Download as pdf or txt
Download as pdf or txt
You are on page 1of 28

PHÂN HOẠCH NGUYÊN

ĐẶNG THU HƯƠNG

THPT CHUYÊN HẠ LONG


Email: dangthuhuong91@gmail.com

Tóm tắt nội dung


Thế nào gọi là "số học-tổ hợp". Thực ra không có một định nghĩa nào cho loại bài toán
đó, nhưng có thể hiểu đó thực chất là các bài toán tổ hợp được làm hấp dẫn thêm bởi một
vài tính chất số học của số tự nhiên. Những bài toán dạng "số học-tổ hợp" được coi như là
nghiên cứu về "cấu trúc tập số tự nhiên" thay vì nghiên cứu về các mối liên hệ đại số và
các tính chất không rời rạc của các số tự nhiên trong bộ môn đại số, giải tích hay số học
khác.
Chủ đề về "số học-tổ hợp" rất rộng trong toán học, bao gồm cả kiến thức tổ hợp và số học,
thậm chí là cả đại số cao cấp. Tuy nhiên mối quan tâm dành cho các dạng toán này hiện
nay còn giới hạn. Để tiện theo dõi, tôi tạm chia đề tài thành 4 phần sau: Phân hoạch trên
tập hợp số nguyên, Phân hoạch và các vấn đề liên quan, Tính chia hết và Các bài toán về
tổng. Trong đề tài này tôi chỉ trình bày một vài mảng lớn và thông dụng trong "số học- tổ
hợp" thường gặp ở những kỳ thi học sinh giỏi chứ không phải tất cả các chủ đề trong "số
học- tổ hợp".

1
Mục lục

1 Phân hoạch tập hợp số nguyên 3

2 Phân hoạch và các vấn đề liên quan 9

3 Tính chia hết 17

4 Các bài toán về tổng 22

2
§1 Phân hoạch tập hợp số nguyên
Một mục khá quan trọng của số học tổ hợp là nghiên cứu về phân hoạch tập hợp số nguyên.
Ví dụ, một kết quả của Schur nêu rằng: "Mọi số nguyên dương k, tồn tại số nguyên n thỏa
mãn: khi các số nguyên từ 1, 2, . . . , n được chia ra thành k tập con, mỗi tập con chứa 3 số
phân biệt sao cho có 1 số là tổng của hai số kia". Hay định lý Van der Waerden như sau:
"Nếu tập tất cả các số nguyên dương được chia ra hữu hạn các tập con, thì tồn tại một tập
nào đó chứa dãy cấp số cộng dài tùy ý". Để tìm thêm được các tính chất mà các tập con
trong phân hoạch nguyên có, thì ta cần tìm hiểu thêm về mối quan hệ giữa các tập con đó.
Phần này ta chỉ giới hạn trong mục phân hoạch trên tập hợp số nguyên và những kỹ thuật
xây dựng, quy nạp, phản chứng, chứng minh trực tiếp đều được sử dụng ở những bài tập
dưới đây.
Bài toán 1: Có thể hay không các số 1, 2, . . . , 100 lập lên đủ 12 dãy cấp số nhân? (Russia, 1995)
Lời giải: Một tập hợp các dãy số {(ai,n )n∈N }i∈I được gọi là phủ kín tập S khi S ⊆ i∈I n∈N {ai,n }.
S S

Ta cố định i ∈ I, một dãy (ai,n )n∈N được gọi là dãy cấp số nhân khi tồn tại số thực
qi thỏa mãn ai,n = ai,0 qin với mọi n ∈ N.
Giả
S sử phản chứng, tồn tại ba số nguyên tố phân biệt pS 1 , p2 , p3 thỏa mãn {p1 , p2 , p3 } ⊆

n∈N {a i,n } với i ∈ I nào đó. Nếu |qi | ∈ {0, 1} thì n∈N {ai,n } ≤ 2. Nếu |qi | > 1
(tương tự với trường hợp 0 < |qi | < 1, và ta cũng có p1 < p2 < p3 ).
Suy ra tồn tại 3 số nguyên dương x1 < x2 < x3 thỏa mãn
x
pj = ai,xj = ai,0 qi j

với mọi j ∈ {1, 2, 3}.


Đặc biệt
  1   1
p2 x2 −x1 p3 x3 −x2
qi = = ,
p1 p2
rõ ràng không thể xảy ra. Tức là mọi dãy
S cấpSsố nhân chứa nhiều nhất 2 số nguyên
tố phân biệt. Đặc biệt kéo theo nếu S ⊆ i∈I n∈N {ai,n } với tập S xác định thì bất
đẳng thức sau đây là đúng:

[ [
|S ∩ P| ≤ {ai,n } ∩ P ≤ 2|I|.
i∈I n∈N

Mệnh đề phủ định sẽ dẫn đến bất đẳng thức sai π(100) ≤ 24 (có 25 số nguyên tố từ 1
đến 100), khi chúng ta đặt S = {1, . . . , 100} và I = {1, . . . , 12}. 
Từ bài toán trên ta nhận thấy dãy 1, 2, . . . , 100 không thể phủ kín bởi 12 dãy cấp số
nhân, vậy 12 dãy có phải điều kiện chặt không? Ta xét bài toán sau:

Bài toán 2: Có thể hay không dãy 2, 3, . . . , 100 được phủ kín bởi 20 dãy cấp số nhân?
Lời giải: Ta chứng minh câu trả lời là không, rõ ràng đây là bài toán mạnh hơn bài toán ban
đầu.
Cố định i ∈ I và giả sử (ai,n )n∈N là một dãy hình học thỏa mãn n∈N {ai,n } ∩ P ≥ 2.
S

Khi đó ta có kết quả sau

[
{ai,n } ∩ Z = 2,
n∈N

đây là một kết quả khá mạnh.


Theo chứng minh bài toán ban đầu, ta có n∈N {ai,n } ∩ P = 2. Ta phải chứng minh
S

khi đó không tồn tại các số nguyên khác được phủ kín bởi dãy số trên.
Xác đinh các số nguyên x1 < x2 như trước thỏa mãn p1 = ai,x1 = ai,0 qix1 và p2 =
ai,x2 = ai,0 qix2 với số nguyên tố p1 < p2 và số thực qi lớn hơn 1. Khi đó qi =

3
1
(p2 /p1 ) x2 −x1 , do đó mọi phần tử của dãy cấp số nhân có dạng nhất định, suy ra
(   z )
[ [ p2 x2 −x1
{ai,n } = p1 .
n∈N
p1
z∈Z∩[−x1 ,∞)

Giả sử với ngược lại tồn tại các số nguyên α 6∈ {p1 , p2 } và y ≥ −x1 thỏa mãn ai,y = α.
Khi đó h
x −x1
p2 2
α= h
x −x1
−1
p1 2
Rõ ràng điều kiện y 6∈ {p1 , p2 } tương đương với h 6∈ {0, x2 − x1 }.
Nếu h lớn hơn x2 − x1 thì phân số là tối giản, do đó không là số nguyên. Nếu 0 < h <
x2 − x1 thì α sẽ có dạng pl1 pm
2 với số hữu tỉ 0 < l, m < 1 tương ứng, do đó cũng không
|h| |h|
+1
là số nguyên. Cuối cùng, nếu h âm, α sẽ có dạng p1 2 1 /p2 2 1 , và cũng không
x −x x −x

thể là số nguyên.
Xét chỉ số i ∈ I thỏa mãn n∈N {ai,n } ∩ P = {p}, với số nguyên tố p nào đó. Và giả sử
S

[
{ai,n } ∩ Z ≥ 2.
n∈N

Khi đó tồn tại số nguyên β 6= p lớn hơn 1 thỏa mãn ai,x1 = β và ai,x2 = p với x1 , x2
1
là số nguyên không âm nào đó. Nó kéo theo qi = (β/p) x2 −x1 , và do đó mọi số hạng
z
của dãy sẽ có dạng p(β/p) x2 −x1 với số nguyên z ≥ −x2 .
Mặt khác, giả sử β = pk với số nguyên k ≥ 1. Khi đó các hoạng tử trong dãy sẽ có
z
dạng pk x2 −x1 , là số nguyên khi và chỉ khi x2 − x1 | z. Điều đó có nghĩa là tập hợp
giao với Z chỉ được tạo bời các phần
 tử có dạng pkn với số nguyên không âm n.
z

Mặt khác, giả sử p ∤ β. Khi đó υp p(β/p) x2 −x1
phải thuộc N, điều này xảy ra khi
và chỉ khi z ∈ {0, x2 − x1 }. Vậy có thể kết luận rằng không có số nguyên nào khác sẽ
bị phủ trong trường hợp này.
Để tính tổng trong trường hợp tốt nhất, dãy cấp số nhân (2n )n≥1 và (3 · 2n )n≥1 sẽ phủ
6 số nguyên trong {2, 3, . . . , 100}. Tương tự, chỉ (5 · 2n )n≥1 và (4 · 2n )n≥1 sẽ phủ nhiều
nhất 5 số nguyên, nhưng dãy (4 · 2n )n≥1 không thể cùng dùng chung với dãy (2n )n≥1 .
Tất cả các số này sẽ phủ nhiều nhất 4 số nguyên phân biệt. (*)
Gọi A là số dãy thỏa mãn mỗi dãy trong đó phủ được nhiều nhất 1 số nguyên tố, và B
là số dãy số mà mỗi dãy trong đó phủ được ít nhất 2 số nguyên tố (đặc biệt có đúng
2 số nguyên là 2 số nguyên tố!). Khi đó A + 2B = 25 và 6A + 2B ≥ 99. Vẽ trên đồ thị
A − B dễ thấy A + B có giá trị nhỏ nhất là 20. 

Chú ý 1.0.1. Số 20 dãy cấp số nhân có thể được chặn tốt hơn bởi 24 dãy cấp số nhân
với nhận xét (*).
Chính xác ta cần dùng ít nhất 36 dãy cấp số nhân để phủ kín được dãy số 1, 2, . . . , 100
(đọc thêm ở [6])

Có nhiều cách phân hoạch sử dụng 36 dãy cấp số nhân để phủ kín 1, 2, . . . , 100. Đây
là một ví dụ phân hoạch sử dụng 36 dãy cấp số nhân như sau:
(1,2,4,8,16,32,64)
(3,6,12,24,48,96)
(5,10,20,40,80)
(7,14,28,56)
(9,18,36,72)
(11,22,44,88)
(13,26,52)
(15,30,60)
(17,34,68)

4
(19,38,76)
(21,42,84)
(23,46,92)
(25,35,49)
(27,45,75)
(50,70,98)
(81,90,100)
là đôi một không giao nhau và đã phủ được 7 + 6 + 5 + 3.4 + 10.3 = 60 số nguyên. Sử
dụng thêm 20 dãy cáp số nhân, mỗi dãy có hai phần tử để phủ kín 40 số nguyên còn
lại. Vậy ta đã sử dụng 16 + 20 = 36 dãy cấp số nhân.
Bài toán 3: Bạn muốn tô màu cho các số trong dãy từ 1 đến 100 sao cho nếu hai số bất kỳ có 1
số là ước của số kia thì 2 số đó được tô khác màu. Hỏi số màu ít nhất cần sử dụng là
bao nhiêu?
Lời giải: Dễ thấy 7 màu là đáp số của bài toán vì 1, 2, 4, 8, 16, 32 và 64 phải được tô các màu
khác nhau. Đó cũng là điều kiện đủ: nếu n có m ước nguyên tố trong phân tích thành
nhân tử, thì n được tô bởi màu thứ m + 1. Màu nhỏ nhất ta sử dụng là 1 (cho 1) và
màu lớn nhất ta sử dụng là 7 (với 64 và 96). 
Bài toán 4: Tập các số nguyên dương được phân hoạch ra hữu hạn các tập con. Chứng minh rằng
với cách phân hoạch S ra thành các tập con S1 , S2 , . . . , Sn có tính chất sau: với mọi
số nguyên dương n ,tồn tại i sao cho Si chứa vô hạn bội của n. (BMC contest, 1999)
Lời giải: Gọi các tập con là S1 , S2 , . . . , Sk . Giả sử phản chứng và ta sẽ đi chứng minh mâu
thuẫn.
Khi đó với mỗi Si tồn tại ni thỏa mãn Si chỉ chứa hữu hạn các bội của ni . Đặt
n = n1 n2 . . . nk ; Khi đó mọi bội của n là một bội của ni và mỗi tập Si có thể chứa
hữu hạn các bội n. Nhưng điều đó có nghĩa là k tập con trên cùng chứa hữu hạn các
bội n, và do chúng là phân hoạch của tập hợp số nguyên dương (chứa vô hạn các bội
của n), nên ta có mâu thuẫn. 
Bài toán 5: Chứng minh rằng tập các số nguyên dương có thể được chia thành vô hạn các tập con
vô hạn sao cho : nếu x, y, z, w đều thuộc 1 tập con, thì x − y và z − w cùng thuộc 1
tập con khi và chỉ khi xy = wz
.
Lời giải: Đặt Ak là tập chứa các số có dạng (2k − 1)2n với n = 0, 1, 2, . . . . Ta sẽ chứng minh
cách chia này thỏa mãn yêu cầu bài toán.
Giả sử x, y, z, w ∈ Ak với x > y, z > w. Ta có x = (2k − 1)2a+b , y = (2k − 1)2a , z =
(2k−1)2c+d ; w = (2k−1)2c . Khi đó x−y = (2k−1)(2b −1)2a ; z−w = (2k−1)(2d −1)2c .
Ta có xy = 2b ; w
z
= 2d . Suy ra xy = w
z
⇐⇒ b = d ⇐⇒ x − y và z − w có cùng ước lẻ lớn
nhất. 
Bài toán 6: Giả sử các số nguyên dương được tô bởi một trong bốn màu: đỏ, xanh, lục và vàng.
Gọi m và n là các số nguyên lẻ mà có giá trị tuyệt đối khác nhau . Chứng minh rằng
tồn tại 2 số được tô cùng màu sao cho hiệu của chúng là một trong những giá trị sau
: m, n, m − n, hoặc m + n. (IMO Proposal, 1999)
Lời giải:

Cách 1: Giả sử ngược lại.


Tô màu cho lưới 2D bằng cách (i, j) phải được tô cùng màu với mi+nj. Bây giờ ta
chọn một đơn vị vuông bất kỳ. Các đỉnh của nó là (i, j), (i+1, j), (i, j+1), (i+1, j+
1) với i, j bất kỳ, 4 điểm đó được lần lượt tô cùng màu với h, h+m, h+n, h+m+n
, với h = mi + nj. Giả sử có 2 trong các số h, h + m, h + n, h + m + n được tô
cùng màu, khi đó hiệu của chúng thuộc một trong 4 giá trị sau m, n, m + n hoặc
m − n. Mâu thuẫn. Do đó các đỉnh của mỗi ô vuông đơn vị được tô bởi các màu
khác nhau.
Giả sử có 3 điểm liền kề nhau trong một hàng của lưới được tô bởi các màu khác
nhau. Không mất tính tổng quát giả sử: R B G. Ở hình vẽ dưới , khi đó ∗ phải
là Y và ta có thể tô hết các điểm kề nhau bởi các màu thích hợp. Khi đó x phải

5
là B, và cứ tiếp tục vậy. Do đó ta có thể mở rộng 3 cột không xác định và thấy
rằng chúng tuần hoàn có chu kỳ là 2 theo hàng. Khi đó dễ thấy các hàng trong
lưới 2D phải tuần hoàn với chu kỳ 2.

R
B G R B G
R B G
→G
Y R→G Y R

x R B G
Nhưng nếu không có 3 điểm kề nhau trong một hàng đã cho được tô các màu
khác nhau thì cột đó phải tuần hoàn với chu kỳ 2, và do đó tất cả các cột trong
mạng lưới có chu kỳ là 2. Do đó hoạc tất cả các hàng có chu kỳ 2, hoặc tất cả các
cột có chu kỳ 2.

Giả sử tất cả các hàng có chu kỳ 2. Không mất tính tổng quát giả sử (0, 0) là R
và (1, 0) là B. Khi đó dễ thấy hàng tiếp theo . . . , (0, 1), (1, 1), (2, 1), . . . được tô
các màu G và Y , hàng tiếp theo nữa . . . , (0, 2), (1, 2), (2, 2), . . . được tô màu R và
B và cứ tiếp tục. Do đó (0, m) phải là G hoặc Y với m là số lẻ. Nhưng (n, 0) phải
là R hoặc B. Nhưng số nguyên mn có cùng màu với (0, m) và (n, 0). Mâu thuẫn.
Tương tự nếu tất cả các cột có chu kỳ là 2. Vậy điều giả sử là sai, hay luôn tồn
tại 2 số được tô cùng màu sao cho hiệu của chúng là một trong những giá trị sau
: m, n, m − n, hoặc m + n.
Cách 2: Dưới đây sẽ là cách giải theo hướng số học cho bài toán!

Giả sử x, y > 0 (Rõ ràng ta có thể làm như vậy do nếu x, y âm thì ta thay thế x
bằng −x và/hoặc y bằng −y). Vì vậy ta chỉ cần xét bài toán với các số nguyên
không âm.
Phủ định mệnh đề cần chứng minh là có một tập S ⊂ N thỏa mãn S, S + x, S +
y, S + x + y là các tập dời nhau, và hợp của chúng là N. Điều đó có nghĩa là
2
1 + t + t2 + . . . = s
(1 + tx )(1 + ty ). Giả sử y < x. Ta có 1+t+t +...
P 
s∈S t 1+ty
=
(1 + t + . . . + t y−1 2y
) + (t + t 2y+1
+ ... + t 3y−1
) + . . . = E.
Khi ta chiaPE cho 1 + tx ta phải thu được một chuỗi chỉ có các hệ số là 0 và 1
(chính là s∈S t ), và điều này sẽ dẫn tới mâu thuẫn như sau: các chuỗi của ta
s

chứa 1+t+. . .+t y−1


, vì y < x nên phải tồn tại k thỏa mãn x ∈ (2ky, (2k +1)y −1]
(do x lẻ). Tuy nhiên tồn tại α ∈ {1, 2, . . . , y − 1} thỏa mãn x + α = (2k + 1)y và
α ∈ S (do x >P y là các số dương nên {1, 2, . . . , y − 1} ∈ S ) , và điều này có nghĩa
là nếu chuỗi s∈S t ) của ta không có hệ số âm nào, khi nhân cả biểu thức với
s

1 + t sẽ chứa t
x (2k+1)y
, nhưng E không có số hạng này, do đó ta có mâu thuẫn.
Từ đó ta có điều phải chứng minh. 
Bài toán 7: Mỗi số nguyên được tô bởi một trong hai màu là đỏ và xanh. Biết rằng mỗi tập hợp
hữu hạn A gồm vài số nguyên liên tiếp, thì chênh lệch giữa số các số được tô màu đỏ
và số các số được tô màu xanh trong A là không quá 1000. Chứng minh rằng tồn tại
một tập hợp gồm 2000 số nguyên liên tiếp mà trong đó có chính xác 1000 số được tô
màu đỏ và 1000 số được tô màu xanh.
Lời giải: Gọi ak là hiệu của các số màu đỏ trừ đi các số màu xanh nằm trong tập {k, k +
1, . . . , k + 1999}. Nếu ak = 0 ta có đpcm. Giả sử không có k thỏa mãn ak = 0, không
mất tính tổng quát giả sử a1 > 0.
Ta thấy |ak − ak+1 | ≤ 2 với mọi k, do sự khác nhau này đượcc xác định bởi màu của
k và k + 2000 . Khi đó nếu ak 6= 0, a1 > 0 và ak là số chẵn thì kéo theo ak > 0 với
mọi k. Xét các số từ 1 đến 2002000. Sự khác nhau giữa số các số đỏ và số các số xanh
trong đoạn này là a1 + a2001 + a4001 + · · · + a20000001 , là 1001 số hạng dương và suy ra
lớn hơn 1000. Điều này mâu thuẫn với giả thiết đã cho. 
Ta xét một bài toán tương tự dạng trên nhưng ở mức độ khó hơn sau:
Bài toán 8: Giả sử 1000 học sinh đứng thành một vòng tròn. Chứng minh rằng tồn tại một số
nguyên k với 100 ≤ k ≤ 300 sao cho trong vòng tròn đó có một nhóm gồm 2k học sinh
kề nhau, mà số học sinh nữ trong k học sinh đầu tiên bằng số học sinh nữ trong k học
sinh cuối cùng. (IMO shortlist 2011)

6
Lời giải: Số học sinh được xếp thứ tự từ 1 đến 1000. Đặt ai = 1 nếu học sinh thứ i là nữ, và
ai = 0 nếu học sinh thứ i là nam. Chú ý các chỉ số xét theo modulo 1000.
n+k−1
X n+2k−1
X
Đặt f (n, k) = ai − ai với mọi số nguyên n và k ∈ [100, 300]. Nếu ta có
i=n i=n+k
f (n, k) = 0 với n, k nào đó, thì ta có điều phải chứng minh. Giả sử f (n, k) 6= 0 với mọi
n, k.
Dễ thấy f (n + 1, k) − f (n, k) ∈ [−2, 2], do f (n + 1, k) − f (n, k) = an+2k + an − 2an+k .
1000
X
Khi đó, f (i, k) = 0 với mọi k, vì mỗi học sinh được đếm k lần mang dấu dương
i=1
và k lần mang dấu âm. Suy ra phải tồn tại n để f (n, 100) > 0 và f (n + 1, 100) < 0
vì ngược lại, các dãy f (i, 100) hoặc tất cả cùng dương hoặc tất cả cùng âm, điều này
là không thể. Dễ thấy rằng f (n, 100) = 1 và f (n + 1, 100) = −1 vì nếu không thì
f (n + 1, 100) − f (n, 100) > 2. Suy ra an+100 = 0 vaf an = an+200 = 1.
Suy ra ta có dãy GXBXG với B là kí hiệu con trai, G là kí hiệu con gái, và X là kí
hiệu một dãy của 99 học sinh mà ở đó số học sinh nữ giữa 2 nhóm là bằng nhau.
Xét f (n − 1, 101). Nếu an−1 = 0 thì f (n − 1, 101) = 0, mâu thuẫn. Suy ra an−1 = 1.
Tương tự, an+201 = 1 từ f (n, 101). Bằng quy nạp, ta có an−k = 1 với mọi k ∈ [0, 200].
Khi đó f (n − 200, 100) = 0, mâu thuẫn. Vậy ta có điều phải chứng minh. 
Bài toán 9: Tập các số nguyên được phân hoạch ra thành các dãy cấp số cộng rời nhau. Chứng
minh rằng tồn tại hai dãy cấp số cộng mà có cùng công sai. (Sasha Schwartz)
Lời giải: Giả sử tập các số nguyên dương bị chia thành hữu hạn các tập S1 , S2 , ..., Sk , mỗi tập
trong đó là một cấp số cộng thỏa mãn giữa chúng không có phần tử chung. Gọi

Si = {ai + ndi với n ∈ N ∪ {0}}, di 6= dj for i 6= j.


Với |z| < 1 xét

X X X
zn = z n + ... + zn
n=1 n∈S1 n∈Sk

tương đương với,

z z a1 z ak
= d
+ ... + .
1−z 1−z 1 1 − z dk
Do di 6= dj , ta có thể giả sử d1 > dj for j 6= 1 và cho z → e2πi/d1 thì hạng đầu tiên
z a1
của tổng trên 1−z d1 tiến tới vô cùng trong khi tất cả các số hạng còn lại trong tổng

đều có giới hạn hữu hạn còn vế trái 1−z


z
hiển nhiên có giới hạn hữu hạn. Do đó ta có
mâu thuẫn và thu được điều phải chứng minh. 
Chú ý: Qua cách chứng minh này, ta có thể khẳng định nếu tập số nguyên được
phân hoạch thành các cấp số cộng rời nhau thì tồn tại 2 dãy cấp số cộng có cùng công
sai và công sai đó là công sai lớn nhất trong tất cả các công sai của các cấp số cộng
trên.
Bài toán 10: Tìm số k nguyên dương lớn nhất có tính chất sau: tập hợp các số nguyên dương phân
hoạch thành k tập con A1 , A2 , ..., Ak sao cho với mọi n ≥ 15, với mọi i ∈ {1, 2, ..., k}
tồn tại hai phần tử khác nhau của Ai có tổng bằng n

Lời giải: Ta bắt đầu với trường hợp k = 2. Rõ ràng có thể phân hoạch tập hợp các số nguyên
dương thành hai tập con A1 = {2n; n ≥ 3} ∪ {1, 2}; A2 = {2n − 1; n ≥ 3} ∪ {3, 4} có
tính chất: mọi số nguyên dương ≥ 7 biểu diễn được dưới dạng tổng hai số thuộc A1 và
tổng hai số thuộc A2 . Khi k = 3, dĩ nhiên việc phân hoạch thành"chẵn, lẻ" như trên
được thay bởi phân hoạch theo modulo 3, và cũng như trước cần thêm vào mỗi lớp
đồng dư modulo 3 một số số đầu tiên để bảo đảm mỗi tập đều biểu diễn được mọi số
lớn hơn hoặc bằng 15 dưới dạng tổng hai số. Có thể chọn phân hoạch sau đây:

7
A1 = {1, 2, 3} ∪ {3m; m ≥ 4}, A2 = {4, 5, 6} ∪ {3m − 1, m ≥ 4}, A3 =
{7, 8, 9} ∪ {3m − 2; m ≥ 4}
Dễ thử lại rằng, phân hoạch trên đây thỏa mãn bài ra. Hơn nữa có thể nhận thấy rằng,
điều kiện biểu diễn được mọi số 15 đối với phân hoạch trên đây đã rất chặt, chẳng
hạn số 14 không thể biểu diễn dưới dạng tổng hai số thuộc A2 hoặc hai số thuộc A3 .
Từ đó dự đoán rằng k = 3 là giá trị lớn nhất có thể để tồn tại phân hoạch thỏa mãn
bài ra.
Ta sẽ chứng minh dự đoán trên, nghĩa là với k ≥ 4 không thể phân hoạch tập các số
tự nhiên thành k tập hợp con thỏa mãn bài ra.
Rõ ràng nếu với k ≥ 4 nào đó mà tồn tại phân hoạch thỏa mãn, thì phân hoạch như
vậy cũng tồn tại với k = 4; chỉ cần lấy phân hoạch A1 , A2 , A3 , A4 ∪ A5 ∪ ... ∪ Ak ta
được phân hoạch gồm 4 tập hợp thỏa mãn bài ra. Như vậy chỉ cần chứng minh không
thể tồn tại phân hoạch gồm 4 tập hợp con thỏa mãn bài ra.
Giả sử tồn tại một phân hoạch như vậy: A1 , A2 , A3 , A4 . Như ta đã thấy trong các ví dụ
khi k = 2, k = 3, các tập hợp Ai phải chứa những số nào đó trong những số tự nhiên
đầu tiên. Xét 10 số nhỏ nhất mà mỗi tập hợp Ai đều phải biểu diễn được: 15,16,...,24.
Mỗi số trong 10 số này đều là tổng của 2 số nào đó thuộc tập hợp B = {1, 2, 3, ..., 23}.
Như vậy, mỗi tập hợp Ai cần chứa ít nhất 5 số thuộc tập hợp B. Do bốn tập Ai rời nhau
mà B chỉ có 23 phần tử nên phải tồn tại tập Aj nào đó chứa đúng 5 số thuộc B, giả sử
đó là các số x1 , x2 , x3 , x4 , x5 . Năm số này biểu diễn được đúng 10 số trong các số từ 15
đến 24, tức là 10 số đó chính là 10 tổng có thể {xk +xl , k 6= l; 1 ≤ k ≤ 15}. Từ đó suy ra

15 + 16 + ... + 24 = 4(x1 + x2 + x3 + x4 + x5 )
vì mỗi số xi tham gia trong đúng 4 cặp số. Đẳng thức trên cho ta mâu thuẫn vì tổng
vế trái là 195, trong khi vế phải chia hết cho 4. 

Bài toán 11: Mỗi số trong tập N = {1, 2, 3, · · · , n − 1}, với n ≥ 3, được tô bởi một trong hai màu
đỏ hoặc đen sao cho :
(i) i và n − i luôn được tô cùng màu và
(ii) Với mọi j ∈ N nguyên tố cùng nhau với n, i và |j − i| sẽ được tô cùng màu với
mọi i ∈ N, i 6= j.
Chứng minh rằng tất cả các số trong N đều được tô cùng màu.
Lời giải: Ta kí hiệu a ∼ b nếu a và b được tô cùng màu và đây là một quan hệ tương đương.
Lấy j ∈ N thỏa mãn gcd(n, j) = 1.
Nếu a < b, j | b − a, thì ta có b = a + kj, với k là số nguyên dương. Khi đó, theo (ii),
ta có b = a + kj ∼ a + (k − 1)j ∼ · · · ∼ a + j ∼ a, nên nếu a ≡ b (mod j) kéo theo
a ∼ b.
Với mọi 0 ≤ c < j ta thấy c ∼ j − c và j − c ∼ n − (j − c) = (c + n) − j. Nhưng
(c + n) − j ≡ c + n (mod j), suy ra c ∼ j − c ∼ (c + n) − j ∼ c + n. xét modulo j ta
có c ∼ c + n ∼ c + 2n ∼ · · · ∼ c + (j − 1)n. Nhưng do gcd(n, j) = 1, nên modulo j của
các phần tử c, c + n, c + 2n, . . . , c + (j − 1)n là hệ thặng dư đầy đủ, do đó ta có các
hoán vị của 0, 1, 2, . . . , j − 1, nên 0 ∼ 1 ∼ 2 ∼ · · · ∼ j − 1, suy ra tất cả các số trong
N đều được tô cùng màu. 
Bài toán 12: Tập các số nguyên dương được chia ra hữu hạn các tập con rời nhau. Chứng minh rằng
tồn tại một tập con ,giả sử là Ai , có tính chất sau: Tồn tại số nguyên dương m thỏa
mãn với mọi k, ta có thể tìm ra các số a1 , a2 , . . . , ak thuộc Ai với 0 < aj+1 − aj ≤ m
cho mọi j, 1 ≤ j ≤ k − 1. (IMO Proposal, 1990)
Lời giải: Gọi tập A ⊂ N là tốt nếu nó thỏa mãn yêu cầu của bài toán nghĩa là tồn tại m thỏa mãn
với mọi k tồn tại a1 , a2 , ..., ak ∈ A thỏa mãn 0 < ai+1 − ai ≤ m, với i = 1, 2, ..., k − 1.
Gọi dãy bị chặn bởi x nếu nó tăng ngặt và 2 số hạng khác nhau liên tiếp của nó sai
khác nhau không quá x.
Bổ đề. Nếu A, B ⊂ N thỏa mãn A ∩ B = ∅ và A ∪ B là tốt, thì có ít nhất một trong 2
tập A và B là tốt.
Do A ∪ B là tập tốt nên tồn tại m sao cho thỏa mãn đề bài. Nếu A không phải là tập
tốt. Vậy với mọi m luôn tồn tại k(m) sao cho với mọi k(m) số thuộc A luôn tồn tại

8
2 số liên tiếp hơn nhau m. Ta đi chứng minh tập B là tập tốt. Do A ∪ B tôt nên xét
k = (k(m) + 1)2 luôn tồn tại a1 < a2 < · · · < ak phần tử thuộc A ∪ B mà hiệu 2 số
liên tiếp bất kỳ luôn nhỏ hơn m. Do dãy này là dãy tốt nên có không quá k(m) phần
tử thuộc A vậy luôn tồn tại 1 dãy con của B có độ dài k(m) sao cho hiệu hai số liên
tiếp bất kỳ luôn nhỏ hơn m (kể cả độ dài lớn hơn k(m)). Vậy B là tập tốt.
Quay trở lại bài toán, gọi A1 ∪ A2 ∪ ... ∪ An là một phân hoạch của tập các số nguyên.
Nếu A1 tốt, thì ta có điều cần tìm. Nếu A1 không tốt, khi đó A2 ∪ A3 ∪ ... ∪ An là tốt.
Nếu A2 tốt thì bài toán được giải quyết còn nếu A2 không tốt, khi đó A3 ∪ ... ∪ An là
tốt, cứ tiếp tục như vậy. Cuối cùng ta được nếu An−1 không tốt, khi đó An là tốt. 
Bài toán 13: Xác đinh rằng có tồn tại hay không số nguyên n > 1 thỏa mãn: Tập các số nguyên
dương có thể được phân hoạch thành n tập con khác rỗng sao cho tổng bất cứ n − 1
phần tử mà lấy mỗi phần tử ra từ một tập trong n − 1 tập con, thuộc tập con còn lại.
(IMO Proposal, 1995)
Lời giải: Trả lời: không.
Rõ ràng không có phân hoạch nào với trường hợp n = 2. Xét n > 2. Gọi một phân
hoạch thỏa mãn đề bài là A1 ∪ A2 ∪ · · · ∪ An . Ta chứng minh nếu a, b, c (không nhất
thiết phân biệt) thuộc cùng một tập con thì a + b và a + c phải cùng thuộc một tập
con.
Giả sử a, b, c thuộc A1 , a + b thuộc A2 và a + c thuộc A3 . Lấy các phần tử của a4 thuộc
A4 , a5 thuộc A5 , . . . , an thuộc An . Khi đó c + (a + b) + a4 + · · · + an phải thuộc A3 .
Nhưng nó bằng b + (a + c) + a4 + · · · + an , tổng này phải thuộc A2 . mâu thuẫn.
Giả sử a + b thuộc A1 và a + c thuộc A3 . Như trước đó ta lấy bất kỳ ai thuộc Ai . Nên
b+a3 +· · ·+an phải thuộc A2 , do đó (b+a3 +...+an )+(a+c)+a4 +· · ·+an = (a+b+
c)+(a3 +· · ·+an )+(a4 +· · ·+an ) phải thuộc A1 . Nhưng c+a3 +· · ·+an phải thuộc A2 ,
do đó (a+b)+(c+a3 +· · ·+an )+a4 +· · ·+an = (a+b+c)+(a3 +· · ·+an )+(a4 +· · ·+an )
phải thuộc A3 . Mâu thuẫn. Chỉ còn lại trường hợp là a + b và a + c thuộc cùng một
tập con.(*)
Lấy ai từ mỗi Ai . Đặt s = a1 + · · · + an . Không mất tính tổng quát giả sử s thuộc
A1 . Đặt bi = s − ai . Ta có thể biểu diễn bi đưới dạng tổng của một phần tử từ mỗi
tập con trừ Ai , nên bi phải thuộc Ai . Ta có ai + bi = s thuộc A1 . Suy ra ai + ai = 2ai
cũng thuộc A1 (theo kết quả (*)).
Giả sử 2m là số dương chẵn bất kỳ. Ta có m thuộc Ai nào đó. Lấy tập bj , thỏa mãn
bj = aj trừ ra bi = m. Lập luận tương tự 2b1 , 2b2 , . . . , 2bn cùng thuộc một tập con.
Nhưng nếu ta lấy j bất kỳ khác i, ta đã có 2bj = 2aj thuộc A1 . Nên 2m cũng phải
thuộc A1 . Nói cách khác, tất cả các số chẵn đều thuộc A1 .
Xét a1 + a3 + · · · + an . Tổng này thuộc A2 . Giữ a3 , a4 , . . . , an cố định và thay a1 lần
lượt bởi các số chẵn. Khi đó ta thấy tất cả các số lẻ lớn hơn a3 + ... + an phải thuộc
A2 . Tương tự bằng cách xét a1 + a2 + a4 + ... + an , ta thấy tất cả các số lẻ lớn hơn
a2 + a4 + ... + an phải thuộc A3 . Mâu thuẫn. Ta có điều phải chứng minh. 

§2 Phân hoạch và các vấn đề liên quan


Xét đồng nhất thức A = B, với A và B là 2 cách đếm khác nhau của cùng một tập hợp các
đối tượng. Ta có thể có bao nhiêu đồng nhất thức như vậy? Một trong các cách tiếp cận là
đếm số phần tử trong A và chứng minh nó có cùng số phần tử trong B. Một cách tiếp cận
khác là tìm song ánh giữa hai tập hợp A và B. Một ví dụ truyền thống cho thấy sự tương
phản giữa 2 cách làm này như sau: số người và số ghế trong hội trường. Với cách thứ nhất,
ta sẽ đếm số người trong phòng và sau đó đếm số ghế trong phòng. Tuy nhiên, theo cách
thứ 2, ta chỉ cần cho mọi người ngồi xuống và đếm số ghế còn thừa hoặc số người không có
ghế ngồi.
Khi chứng minh các đẳng thức có liên quan đến sự phân hoach các số nguyên, có bài thì dễ
tiếp cận với cách thứ nhất (hàm sinh), cũng có những bài thì lại dễ tiếp cận với phương án
thứ hai (chứng minh song ánh), và còn có cả những bài cùng dễ hoặc cùng khó với cả hai
phương án trên. Trong bài viết này ta sẽ xét các ví dụ xảy ra theo ba trường hợp như đã

9
nói.
Trước hết ta cần tìm hiểu khái niệm của phân hoạch của một số nguyên dương.
Định nghĩa 2.1. Nếu n là số nguyên dương, một phân hoạch của n là dãy các số nguyên
dương không tăng p1 , p2 , ..., pk có tổng là n. Khi đó mỗi pi được gọi là thành phần của phân
hoạch. Ta định nghĩa hàm p(n) là số các phân hoạch của n

Ví dụ p(5) = 7 vì ở đây ta có 5 phân hoạch của n = 5:

5 = 5
= 4+1
= 3+2
= 3+1+1
= 2+2+1
= 2+1+1+1
= 1+1+1+1+1+1+1
Ta có p(n) = 0 với n là số nguyên âm và quy ước p(0) = 1
Phân hoạch nguyên được đề xuất đầu tiên bởi Euler. Sau nhiều năm với rất nhiều câu hỏi
hay và khó về nó là xác định tính chất giới hạn hay gọi là tiệm cận của p(n) với n đủ
lớn. Vấn đề này cuối cùng được giải quyết gần như hoàn chỉnh bởi Hardy, Ramanujan và
Rademacher [3] [4]. Tuy nhiên câu hỏi sau: "Tìm các tiêu chuẩn để xác định khi nào p(n) là
số lẻ hay chẵn?" vẫn là một cậu hỏi chưa được giải đáp mặc dù chúng ta có thể tính được
giá trị của p(n) với giá trị n đến vài tỉ.

Các biểu đồ Ferrers về phân chia các số nguyên là một công cụ hữu ích để có thể hình
dung ra sự phân chia và có thể chúng minh vài đẳng thức liên quan đến sự phân hoạch các
số nguyên dương.
Biểu đồ Ferrers tương ứng với một cách phân hoạch λ là một biểu diễn bằng hình vẽ về
phân hoạch λ. Để xây dựng một biểu đồ Ferrers về phân hoạch λ = (λ1 , λ2 , . . . , λk ), đơn
giản là ta đặt các hàng của λi+1 ngay bên trên hàng λi và các hàng được căn trái, với mọi
i = 1, 2, . . . , k − 1. Ví dụ biểu đồ Ferrers của phân hoạch λ = (6, 4, 3, 1, 1) như sau

Các công cụ chính chúng ta sử dụng để đếm số lượng các phân hoạch là dựa vào dãy
cấp số nhân:
1 1
= 1 + q + q2 + q3 + · · · = 1 + q n + q 2n + q 3n + · · ·
1−q 1 − qn

Chúng ta có thể hình dung dãy cấp số nhân trên theo biểu đổ Ferrers theo hàng như sau:
1 1
=1+ + + + ··· =1+ + + + ···
1− 1−

hoặc theo cột như sau:


1 1
=1+ + + + ··· =1+ + + +···
1− 1−

Sau đây là ví dụ sử dụng biểu đồ Ferrers trong chứng minh các đẳng thức liên quan đến
phân hoạch các số tự nhiên.
Bài toán 1: Số các phân hoạch của số nguyên n có thành phần lớn nhất bằng k bằng số phân
hoạch của n có k thành phần.

10
Lời giải: Để chứng minh định lý này, ta bắt đầu bằng giản đồ Ferrers và nhận thấy rằng khi ta
hoán đổi các hàng và cột với nhau ta được tương ứng 1 − 1 giữa hai loại phân hoạch
Ví dụ cho phân hoạch λ = (4, 3, 2, 2, 1)trường hợp n = 12 và k = 5

tương đương với


Ta định nghĩa hàm p(n, k) là số các phân hoạch của n có thành phần lớn nhất là k
Bây giờ ta sẽ chỉ ra một hàm sinh Euler cho dãy {p(n)}∞
n=0 . Nói cách khác, ta đi tìm

dạng đẹp cho hàm n
P
p(n)x
n=0
Xét
(1 + x + x2 + x3 + ...)(1 + x2 + x4 + x6 + ...)(1 + x3 + x6 + ...)(1 + x4 + x8 + ...)...(1)


Ta cần mở rộng các tích này để có được kết quả mong muốn p(n)xn này. Điều
P
n=0
quan trọng để hiểu tại sao điều này là đúng thì ta đi vào vài ví dụ cụ thể sau . Xét
hệ số của x3 . Bằng cách chọn x trong nhân tử đầu tiên,x2 trong nhân tử thứ hai, và
hằng số 1 trong các nhân tử còn lại ta được hệ số của x3 bằng 1. Tương tự, ta chọn
x3 từ nhân tử thứ 3 và 1 trong các nhân tử còn lại, ta được hệ số của x3 cũng bằng
1. Vậy nó liên quan thế nào đến sự phân hoạch của số nguyên.
Xét đơn thức trong nhân tử thứ i trong (1): 1 + xi + x2i + x3i + ... thể hiện số lần xuất
hiện của thành phận i trong phân hoạch số nguyên. Cụ thể, nếu ta chọn đơn thức xci i
từ nhân tử thứ i, thì i xuất hiện ci lần trong phân hoạch. Với mỗi cách chọn đơn thức
cho ta một thành phần trong phân hoạch ứng với hệ số xn và nói chung, khi xn được
tạo ra từ 1 hạng tử trong khai triển (1) như sau x1c1 .x2c2 .x3c3 ... thì tương ứng với cách
viết n dưới dạng n = 1c1 + 2c2 + 3c3 + ... với ci ≥ 0. Rõ ràng đây là một cách biểu diễn
một phân hoạch nguyên của n. Ví dụ, phân hoạch của 25 = 6+4+4+3+2+2+2+1+1
có thể viết dưới dạng 25 = 1(2) + 2(3) + 3(1) + 4(2) + 5(0) + 6(1). Suy ra ta có tương
ứng 1 − 1 giữa cách chọn các đơn thức để có tích là xn trong các nhân tử từ khai triển
(1) và các phân hoạch nguyên của n
Quay lại (1), ta thấy mỗi số hạng trong tích là một dãy cấp số nhân. Tích đó có thể
viết lại là:
1 1 1
....
1 − x 1 − x2 1 − x3
Tuy nhiên chúng ta không quan tâm đến việc dãy trên có hội tụ hay không mà chỉ
quan tâm đến số mũ của x trong khai triển.
Qua những nhận xét trên ta rút ra được định lý Euler như sau:
Định lý 2 (Euler)
1 1 1 ∞
def
p(n)xn .
P
ε(x) = 2 3
.... =
1−x1−x 1−x n=0
Bây giờ ta xét một số bài tập sau.

Bài toán 2: Số các phân hoạch nguyên của n không có thành phận nào bằng 1 là p(n) − p(n − 1).
Lời giải: Cho f (n) là số phân hoạch nguyên của n không có thành phần nào bằng 1. Dễ thấy
đơn thức chọn từ nhân tử nhân tử (1 + x + x2 + x3 + ...) biểu hiện số các số 1 trong
phân hoạch. Do đó ta có thể chỉ chọn hằng số 1 từ nhân tử trên, và ta thu được hàm

11
sinh như sau:
∞ 1 1
f (n)xn
P
= 1. ....
n=0 1 − x2 1 − x3
= (1 − x)ε(x)
∞ ∞
p(n)xn − p(n − 1)xn .
P P
=
n=0 n=1

(p(n) − p(n − 1))xn
P
=
n=0
Bằng cách nối các hệ số bậc của xn tương ứng trong cả hai vế ta thu được điều phải
chứng minh. 
Phần này bạn đọc tự chứng minh, bằng cách chỉ ra song ánh giữa 2 đồng nhất thức.
Đây là chủ đề tổng quát sẽ xuất hiện trong các ví dụ tiếp theo: ta chứng minh đồng
nhất thức các phân hoạch nguyên qua hàm sinh, tuy nhiên để hiểu rộng hơn, ta cần
tìm một lời giải bằng song ánh.
Với các bài tập khác, giả sử có 2 tập số nguyên dương S và T đã biết. Khi đó ta sẽ
xác định hàm sinh của số phân hoạch nguyên của n có tất cả bộ phận nằm trong S
và của số phân hoạch nguyên của n có tất cả bộ phận nằm trong T .

Bài toán 3: Chứng minh rằng mọi số nguyên dương chỉ biểu diễn đươc duy nhất dưới dạng tổng
các lũy thừa của 2
∞ j
Lời giải: Xét (1 + x)(1 + x2 )(1 + x4 )(1 + x8 )... = (1 + x2 ).
Q
j=0
Đây là một biến thể của hàm Euler ε(x). Nhận thấy 1 + x là bắt đầu của 1 + x + x2 +
...,1 + x2 là bắt đầu của 1 + x2 + x4 + ..., và 1 + x4 là bắt đầu của 1 + x4 + x8 + ...,...
Suy ra ta có thể đếm các phân hoạch nguyên của n có tất cả các bộ phận là lũy thừa
của 2, và hơn thế nữa, mỗi lũy thừa của 2 có thể xảy ra nhiều nhất 1 lần. Vì
∞ 1
j
(1 + x2 ) = = 1 + x + x2 + x3 + ...
Q
j=0 1−x
Phương trình này có thể chứng minh mọi số nguyên dương chỉ biểu diễn đươc duy
nhất dưới dạng tổng các lũy thừa của 2. 
Bây giờ ta xét hệ thức
∞ 1 ∞
f (n)xn
Q P
2j+1
=
j=0 1 − x n=0
Trong trường hợp này, f (n) đếm số phân hoạch nguyên của n có các thành phần là số
lẻ.
Ta đi xét ứng dụng của hàm sinh này qua bài toán sau:
Bài toán 4: Chứng minh rằng số cách phân hoạch số nguyên dương n thành tổng các số khác nhau
đôi một bằng số cách phân hoạch n thành tổng các số lẻ.
Lời giải: Chứng minh bằng hàm sinh: Ta xét định lý này trong trường hợp n = 5
Phân hoạch Lẻ các thành phần Các thành phần đôi một phân biệt
5 * *
4+1 *
3+2 *
3+1+1 *
2+2+1
2+1+1+1
1+1+1+1 *
Ta thấy định lý đúng với n = 5; bây giờ ta sẽ chứng minh định lý bằng hàm sinh.
Hàm sinh biểu diễn phân hoạch thành các thành phần khác nhau đôi một là (1 +
x).(1 + x2 ).(1 + x3 )... trong đó hàm sinh biểu biễn phân hoạch thành các thành phần
1 1 1
là số lẻ là . . . . . . Mà ta có
1 − x 1 − x3 1 − x5

12
1 − x2 1 − x4 1 − x6
(1 + x).(1 + x2 ).(1 + x3 )... = . . ...
1 − x 1 − x2 1 − x3
1 1 1
= . . ...
1 − x 1 − x3 1 − x5
Đây là mà một ví dụ có chứng minh dễ dàng bằng cách sử dụng hàm sinh. Tuy nhiên
có nhiều người lại thích sử dụng phương pháp song ánh. Trong trường hợp này ta cần
một chứng minh bằng song ánh với mọi phân hoạch nguyên thành các thành phần là
số lẻ tới mọi phân hoạch số nguyên thành các thành phần đôi một phân biệt. Quan
điểm đó cho ta một ánh xạ.
Chứng minh bằng song ánh Euler: Một phân hoạch nguyên có các thành phần khác
nhau có thể viết dưới dạng sau:
n = d1 + d2 + ... + dk
với mỗi số nguyên di được biểu diễn duy nhất dưới dạng lũy thừa của 2 nhân với 1 số
lẻ. Suy ra n = 2a1 O1 + 2a2 O2 + 2a3 O3 + ... + 2ak Ok , với Oi là số lẻ. Nếu ta nhóm các
số lẻ lại với nhau ta sẽ được biểu thức:
n = (2α1 + 2α2 + ...).1 + (2β1 + 2β2 + ...).3 + (2γ1 + 2γ2 + ...).5 + ...
= µ1 .1 + µ3 .3 + µ5 .5 + ...
Trong mỗi chuỗi (2α1 + 2α2 + ...), các αi phân biệt do đây là một phân hoạch thành
các thành phần khác nhau. Bây giờ ta xét các phân hoạch nguyên của n có các các
thành phần là số lẻ qua phương pháp song ánh, của phân hoạch nguyên đã cho từ các
thành phần phân biệt. Đó là phân hoạch nguyên chứa µ1 số 1, µ3 số 3 ,. . . . Và phép
song ánh được hình thành. Ta có điều phải chứng minh. 
Để mô tả chứng minh này ta xét phân hoạch nguyên của 5 có các bộ phận phân biệt:
5 = 3 + 2. Xét cùng song ánh của nó, trong phân hoạch nguyên của 5 gồm các bộ phận
lẻ? Để trả lời vấn đề này ta tiến hành như chứng minh trên.
5 = 3+2
= 20 .3 + 21 .1
= 21 .(1) + 20 (3)
= 2 lần số 1 + 1 lần số 3
= 3+1+1
Sau đây là một số bài tập về phân hoạch mà sẽ sử dụng phương pháp song ánh và
hàm sinh thích hợp.

Bài toán 5: Cho m và n là số nguyên dương với n > 21 m(m + 1). Chứng minh rằng số cách phân
hoạch n thành m thành phần khác nhau bằng số cách phân hoạch của n − 21 m(m + 1)
thành nhiều nhất m thành phần. (Korea, 1995)
Lời giải: Với bài này ta sẽ sử dụng phép song ánh. Với mỗi cách phân hoạch n thành m thành
phần khác nhau giả sử là (n1 , n2 , . . . , nm ) với n1 < n2 < n3 < · · · < nm ta biến đổi
thành (n1 − 1, n2 − 2, . . . , nm − m) ứng với một cách phân hoạch n − 12 m(m + 1) thành
nhiều nhất m thành phần vì ta dễ thấy ni − i ≥ 0 với mọi i = 1, 2, . . . , m. Ta có điều
phải chứng minh. 
Bài toán 6: Cho n là số nguyên dương. Chứng minh rằng số cách phân hoạch n bằng số cách phân
hoạch 2n thành tổng n phần.
Lời giải: Ta sẽ tổng quát hóa bài toán là đi chứng minh số cách phân hoạch n thành nhỏ hơn
hoặc bằng m thành phần bằng số cách phân hoạch m + n thành tổng của chính xác
m thành phần.
Đặt Pnm là cách phân hoạch n thành chính xác m phần.
Giờ xét tập hợp E là tập các cách phân hoạch của n thành nhỏ hơn hoặc bằng m
thành phần, ta có thể coi như là bộ m phần tử. Tương tự gọi gọi tập E1 là tập các
cách phân hoạch m + n thành tổng của chính xác m . Trên E ta xét hàm số
f :E → E1
(a1 , a2 , a3 , ..., ak , 0, 0, 0..., 0) → (a1 + 1, a2 + 1, .., ak + 1, 1, 1....1) .
Dễ thấy f là song ánh , vì mọi bộ m phần tử của E1 là ảnh của bộ m phần tử của
E và mọi bộ m phần tử của E được biến đổi thành duy nhất bộ m phần tử của E1 .
Cho nên |E| = |E1 | . Ta có Pn1 + Pn2 + ... + Pnm =Pn+m m
, p1n = 1 = Pnn . Ta đã chứng
minh được bài toán tổng quát . Bài toán cần chứng minh chỉ là trường hợp n = m .

13
Bài toán 7: Quy ước P (n) là số cách phân hoạch n thành tổng các số nguyên dương và đặt Q(n)
là số cách phân hoạch 2n thành tổng các số nguyên dương sao cho không có chính xác
n phần tử. Chứng minh rằng
n−1
X
Q(2k ) = P (2n ) − 1
k=0

Lời giải: Chúng ta sẽ đi chứng minh bằng quy nạp.


Với n = 1 ta đi chứng minh Q(1) = P (2)−1. Rõ ràng là đúng, vì Q(1) = 1 và P (2) = 2.
Giả sử đẳng thức cần chứng minh đúng với n, ta cần chứng minh đẳng thức đó đúng
với n + 1 nghĩa là cần chứng minh
n
X
Q(2k ) = P (2n+1 ) − 1
k=0

Trước hết ta sẽ chứng minh điều sau: Q(2n ) = P (2n+1 ) − P (2n ).


Ta nhận thấy P (2n+1 ) đếm tất cả các phân hoạch của 2n+1 thành các thành phần
nguyên dương. Mặt khác, Q(2n+1 ) đếm tất cả các phân hoạch của 2n+1 sao không
có chính xác 2n thành phần. Cho nên ta sẽ cần phải chứng minh số phân hoạch của
2n+1 chính xác 2n thành phần nguyên dương bằng P (2n ). Điều này hoàn toàn đúng
vì tồn tại một song ánh giữa 2 tập phân hoạch trên bằng cách cộng hoặc trừ 1 vào
từng thành phần và chú ý rằng mọi phân hoạch được tính trong P (2n ) có tối đa 2n
thành phần. Ví dụ phân hoạch của 8 = 3 + 2 + 2 + 1 ra thành 4 phần tương ứng với
phân hoạch của 4 = 2 + 1 + 1 + 0. Ta có điều phải chứng minh. 
Bài toán 8: Với mỗi số nguyên dương m, ký hiệu C(m) là số nguyên dương k lớn nhất để tồn tại
tập hợp S gồm m số nguyên sao cho mỗi số nguyên từ 1 đến k đều thuộc S, hoặc là
tổng của hai số thuộc S (không nhất thiết khác nhau).
Ví dụ: C(3) = 8; S = 1, 3, 4
Chứng minh bất đẳng thức sau đây
m(m + 6) m(m + 3)
≤ C(m) ≤
4 2
Lời giải: Điều kiện bài toán gợi cho ta thấy cần phải tính số phần tử của tập hợp S ∪ (S + S),
trong đó
S + S = {x + y|x, y ∈ S}
Nếu với mỗi tập hợp A, ký hiệu qua |A| số phần tử của nó, thì ta có bất đẳng thức
hiển nhiên sau:
|S ∪ (S + S)| ≤ |S| + |S + S|
Mỗi phần tử của S + S nhận được bằng cách lấy tổng một cặp số (bằng nhau hoặc
khác nhau) của S, suy ra bất đẳng thức sau:
2 1
|S + S| ≤ |S| + C|S| = |S|(|S| + 1)
2
Trong hai bất đẳng thức cần chứng minh , bất đẳng thức bên phải tương đối dễ thấy.
Số C(m) là số nguyên dương k lớn nhất sao cho tồn tại tập S gồm m số nguyên dương
thỏa mãn
{1, 2, ..., k} ∈ S ∪ (S + S)
Từ chứng minh trên ta có
1
k ≤ |S ∪ (S + S)| ≤ |S|(|S| + 3)
2
Từ đó suy ra
m(m + 3)
C(m) ≤
2
Ta chứng minh bất đẳng thức bên trái. Trước tiên tập hợp S cần tìm chứa một số số
tự nhiên liên tiếp nào đó {1, 2, ..., t|t ≤ m}. Với những số này, tập hợp S ∪ (S + S)
chứa các số 1, 2, ..., 2t. Như vậy, để biểu diễn được những số tiếp theo, cần thêm số
2t + 1. Các số 1, 2, ..., t, 2t + 1 cho phép biểu diễn các số tự nhiên cho đến số 3t + 1,
vì thế muốn biểu diễn các số lớn hơn, ta cần thêm số 3t + 2. Lý luận trên đây chỉ ra
rằng tập hợp S gồm m phần tử cần tìm có dạng
{1, 2, ..., t} ∪ {(k + 1)t + k; k = 1, 2, ..., m − t}

14
Khi đó dễ thấy rằng
{1, 2, ..., m + (m + 1)t − t2 } ∈ S ∪ (S + S)
Ta cần tìm giá trị t < m sao cho số các số biểu diễn được là lớn nhất, tức là tìm t sao
cho m + (m + 1)t − t2 đạt cực đại. Dễ chứng minh rằng đại lượng này đạt cực đại khi
m+1 m(m + 6)
t=[ ], và giá trị cực đại là [ ]
2 4
Từ đó ta có điều phải chứng minh. 

Bài toán 9: Cho n là số nguyên dương. Giả sử các ước nguyên dương của nó có thể được phân
hoạch thành các cặp tức là được chia thành các nhóm có 2 phần tử thỏa mãn tổng
mỗi cặp là một số nguyên tố. Chứng minh rằng những số nguyên tố đó phân biệt và
không có số nào trong chúng là ước của n.(Benelux Mathematical Olympiad 2016)
Lời giải: Gọi (da1 , db1 ), (da2 , db2 ), · · · , (dak , dbk ) là tất cả các ước nguyên dương của nvà được
phân hoạch thỏa mãn đề bài. Gọi dai + dbi = pci với mọi 1 ≤ i ≤ k. Hiển nhiên pci ≥ 3
và là số lẻ cho nên một trong 2 số dai , dbi phải là số lẻ, số còn lại phải là chẵn . Điều
đó dẫn đến n có chính xác k ước chẵn và k ước lẻ. Gọi n = 2r h thì n có chính xác
τ (h) ước lẻ cho nên τ (n) = 2τ (h) Suy ra r = 1.  
Xét ước nguyên tố lẻ p của n mà pl |n thì ta sẽ có τ pnl ước nguyên tố cùng nhau với
 
p và có l · τ pnl ước mà chia hết cho p. Rõ ràng là không thể xảy ra trường hợp tồn
tại p > 1 sao cho p | dai , dbi với mọi cặp (dai , dbi ), hoặc chỉ có 1 trong
  2 số trên chia
hết cho p hoặc cả 2 số đều không chia hết cho p. Từ đó ta có l · τ n
pl
≤ τ pnl dẫn
đến l = 1. Vì vậy n = 2p1 p2 · · · pm với m ∈ Z+ .
Từ đây ta có thể nhận thấy nếu dai = n thì dbi = 1, nếu dai = 2p1 p2 · · · pm−1 thì
dbi = pm (nó không thể bằng 1 vì ta đã biết rằng 1 cùng cặp với n), ... , và nếu
dai = 2p1 · · · pm−2 thì dbi = pm−1 pm (nó không thể một trong 2 số pm−1 , pm , 1 vì các
số này đã thuộc cặp trước đó rồi), .... Từ đó ta có thể suy ra mọi cặp (dai , dbi ) đều có
dạng (k, n/k) với k là ước của is n.
Bây giờ để chứng minh bài toán thì ta giả sử phản chứng nghĩa là tồn tại 2 cặp khác
nhau (k, n/k) và (l, n/l) thỏa mãn k + nk = l + nl = p suy ra (l − k)(n − lk) = 0, hay
là (k, n/k) = (l, n/l), mâu thuẫn.
 Vậy các số nguyên tố pci này phải phân biệt.
Hiển nhiên rằng gcd k + nk , n = 1 vì gcd k, nk = 1 (do n = 2p1 p2 · · · pm ). Nghĩa là


không có số nguyên tố pci nào là ước của n. Ta có điều phải chứng minh. 
Bài toán 10: Tìm tất cả các cách phân hoạch tập {1, 2, .., n} thành 3 tập con A, B, C sao cho không
có 2 tập nào trong số các tập sau A + B , B + C and C + A có chung phần tử .
Lời giải: Dễ thấy |X +Y | ≥ |X|+|Y |−1. Cho nên 2n−3 = |[3; 2n−1]| ≥ |A+B|+|B +C|+|C +
A| ≥ (|A| + |B| − 1) + (|B| + |C| − 1) + (|C| + |A| − 1) = 2(|A| + |B| + |C|) − 3 = 2n − 3.
Từ đó ta suy ra hợp các tập A + B, B + C, C + A bằng chính xác đoạn [3, 2n − 1].
Không mất tính tổng quát , giả sử 1 ∈ A, 2 ∈ B. Xét k phần tử bé nhất của C. Giả
sử k ≥ 4. Nên các số 3, . . . , k không thể nằm trong A + C và B + C, cho nên sẽ thuộc
A + B. Do 4 thuộc A + B nên 3 ∈ B, do 5 phải thuộc A + B nên 4 ∈ B, . . . , và cứ
tiếp tục vậy thì [2, k − 1] ⊂ B. Hơn nữa chúng ta biết rằng k + 2 = 2 + k = 1 + (k + 1).
Do 2 + k ∈ B + C, cho nên k + 1 không thể thuộc B hoặc C vì như vậy 1 + (k + 1)
thuộc A + B hoặc A + C suy ra k + 1 phải nằm trong tập chứa 1, tức là A. Do
k + 3 = 2 + (k + 1) = 3 + k, (B + C) ∩ (A + B) chứa 3 + k, mà các tập này phải không
giao nhau , vô lý. Vậy k = 3 ∈ C.
Từ đây chúng ta đi chứng minh bằng quy nạp: nếu l ≡ 1( mod 3), thì l ∈ A, nếu
l ≡ 2( mod 3), thì l ∈ B, nếu l ≡ 0( mod 3), thì l ∈ C.
Quả thật mệnh đề trên đúng với l ≤ 3. Giả sử mệnh đề đúng với mọi l ≤ m. Ta cần
chứng minh mệnh đề đúng với l = m + 1.
Ta có 2m−1 = m+(m−1) = (m+1)+(m−2). Chúng ta biết rằng các số m−2, m−1, m
nằm ở các tập khác nhau trong 3 tập A, B, C, cho nên m + 1 phải thuộc trong tập có
chứa m − 2, điều này nghĩa là mệnh đề được chứng minh cho trường hợp l = m + 1.
Vậy chúng ta chỉ có 1 cách phân hoạch duy nhất thỏa mãn đề bài là phân hoạch theo
số dư của modulo 3. 

15
Bài toán 11: Cho Z và Q là tập các số nguyên và các số hữu tỷ theo thứ tự lần lượt. a) Tồn tại
hay không cách phân hoạch Z thành 3 tập con khác rỗng A, B, C sao cho các tập
A + B, B + C, C + A không giao nhau?
b) Tồn tại hay không cách phân hoạch Q thành 3 tập con khác rỗng A, B, C sao cho
các tập A + B, B + C, C + A không giao nhau?
Với X +Y được ký hiệu là tập {x +y : x ∈ X, y ∈ Y }, với X, Y ⊆ Z và X, Y ⊆ Q.(IMO
Shortlist 2012)
Lời giải: (a) Câu trả lời là có tồn tại phân hoạch Z thành 3 tập con khác rỗng A, B, C sao cho
các tập A + B, B + C, C + A không giao nhau. Gọi

A = {x | x ≡ 0 (mod 3)}, B = {x | x ≡ 1 (mod 3)}, C = {x | x ≡ 2 (mod 3)}


Thì

A+B = {x | x ≡ 1 (mod 3)}, A+C = {x | x ≡ 2 (mod 3)}, B+C = {x | x ≡ 0 (mod 3)}


là các tập không giao nhau thỏa mãn đề bài.
Hơn nữa, chúng ta đi chứng minh đây là cách phân hoạch duy nhất (không tính các
hoán vị của chúng) . Vì A, B, C là các tập khác rỗng, nên tồn tại a, b, c thỏa mãn
a ∈ A, b ∈ B, c ∈ C.
Nếu a + 1 ∈ A. Thì (a + 1) + b ∈ A + B. Nếu b + 1 ∈ C, thì a + (b + 1) ∈ C, vô lý. Vậy
b+1∈ / C. Tương tự ta thu được c + 1 ∈ / B.
Nếu b + 1 ∈ A và c + 1 ∈ A, thì b + (c + 1) ∈ A + B và c + (b + 1) ∈ A + C, vô lý. Nên
ít nhất một trong hai điều sau xảy ra b + 1 ∈ B, c + 1 ∈ C. Không mất tính tổng quát
giả sử b + 1 ∈ B. Thì c + (b + 1) ∈ B + C. Nếu c + 1 ∈ A, thì b + (c + 1) ∈ A + B, vô
lý. Do c + 1 ∈ / A =⇒ c + 1 ∈ C. Tương tự như vậy ta sẽ có với mọi k ∈ Z
/ B, c + 1 ∈
thì a + k ∈ A, b + k ∈ B, c + k ∈ C. Giả sử b > a ta có a + (b − a) ∈ A, vô lý.
Tương tự như cách chứng minh trên cho trường hợp a + 2 ∈ A, với mọi k ∈ Z ta có
a + 2k ∈ A, b + 2k ∈ B, c + 2k ∈ C. Theo định lý Dirichlet, tồn tại ít nhất hai trong
ba số a, b, c có cùng tính chẵn lẻ. Không mất tính tổng quát giả sử a, b đều chẵn. Giả
sử b > a thì a = 2x, b = 2y =⇒ 2x + 2(y − x) ∈ A, vô lý.
Nếu a + 3 ∈ / A, không mất tính tổng quát giả sử a + 3 ∈ B. Khi đó a + c + 3 ∈
B + C =⇒ c + 3 ∈ A. Do đó b + c + 3 ∈ A + B, nghĩa là b + 3 ∈ C. Khi đó
a + c + 6 ∈ A + B, a + b + 6 ∈ B + C. Vậy a + 6 ∈ C. Tương tự, c + 6 ∈ B, b + 6 ∈ A.
Tương tự ta sẽ có a + 9 ∈ A, b + 9 ∈ B, c + 9 ∈ C. Giả sử a + 1 ∈ B và do a + 1 + a + 2 =
a + a + 3 ∈ A + B suy ra a + 2 ∈ A, vô lý ta đã chứng minh trên. Giả sử a + 1 ∈ C và do
a+1+a+2 = a+a+3 ∈ A+B suy ra a+2 ∈ C. Tương tự ta lại có b+1 ∈ A, b+2 ∈ A,
c + 1 ∈ B, c + 1 ∈ B. Tuy nhiên (a + 2) + b ∈ C + B, (a + 1) + (b + 1) ∈ C + A. Mâu
thuẫn.
Cho nên với mọi a ∈ A, ta có a + 3 ∈ A, a + 1, a + 2 ∈ / A. Từ đó ta đã chỉ ra chỉ tồn
tại duy nhất cách phân hoạch tập số nguyên dương thỏa mãn đề bài.
b) Giả sử có một phân hoạch Q các tính chất thỏa mãn đề bài. Ta chọn ra ba số hữu
tỷ ri = pqii từ ba tập A, B, C, i = 1, 2, 3, và tập N = 3q1 q2 q3 . Cho S ⊂ Q là tập các số
hữu tỷ với mẫu số là N ( có thể tối giản hay không). Tập này được tạo ra bằng cách
nhân mỗi số tự nhiên với 1 hằng số N1 , cho nên tập này là tập đóng dưới phép cộng
và phép trừ. Hơn nữa, nếu ta coi mỗi k ∈ Z như là phân số N k
∈ S thì S có thể được
coi như tập số nguyên Z dưới phép cộng. Các số ri thuộc S vì
3p1 q2 q3 3p2 q3 q1 3p3 q1 q2
r1 = , r2 = , r3 = .
N N N
Do phân hoạch Q = A ∪ B ∪ C của Q dẫn đến phân hoạch S = A′ ∪ B ′ ∪ C ′ của S với
A′ = A ∩ S, B ′ = B ∩ S, C ′ = C ∩ S. Dễ thấy A′ + B ′ , B ′ + C ′ , C ′ + A′ không giao
nhau đôi một cho nên cách phân hoạch này sẽ thỏa mãn đề bài.
Do tính duy nhât của phân hoạch Z thỏa mãn đè bài nên các tập A′ , B ′ , C ′ là các
lớp đồng dư theo modulo 3, mà được nhân với hằng số N1 . Mà ta dễ thấy các số có
dạng N3 sẽ thuộc cùng 1 tập trong 3 tập: A′ , B ′ hoặc C ′ . Mà ta lại có r1 , r2 , r3 đều có
dạng N3 cho nên 3 số đó phải thuộc cùng 1 tập trong 3 tập A′ , B ′ hoặc C ′ . Tuy nhiên

16
r1 , r2 , r3 lại thuộc 3 tập khác nhau A′ , B ′ , C ′ vì chúng ta chọn ra từ 3 tập khác nhau
A, B, C. Điều giả sử vô lý. Vậy không thể có cách phân hoạch Q có các tính chất thỏa
mãn đề bài. 
Bài toán 12: Một cách phân hoạch π của một số nguyên dương n ≥ 1, chúng ta định nghĩa là cách
biểu diễn n thành tổng của một hay một vài số nguyên dương mà các hạng tử trong
tổng được xếp theo thứ tự không giảm . (Ví dụ, nếu n = 4, thì phân hoạch π của 4 là
1 + 1 + 1 + 1, 1 + 1 + 2, 1 + 3, 2 + 2, và 4). Gọi ℘[n] là tập hợp các cách phân hoạch π
của một số nguyên dương n ≥ 1.
Với mọi cách phân hoạch π, ta định nghĩa A(π) là số các số 1 xuất hiện trong π, và
định nghĩa B(π) là số các số nguyên phân biệt xuất hiện trong π. (Ví dụ: nếu n = 13
và π là cách phân hoạch 1 + 1 + 2 + 2 + 2 + 5, thì A(π) = 2 và B(π) = 3).
Chứng minh rằng với mọi n cố định, tổng của A(π) chạy khắp các cách Pphân hoạch π
của n bằng tổng của B(π) chạy khắp các cách phân hoạch π của n hay π∈℘[n] A(π) =
π∈℘[n] B(π) .(1986 USAMO )
P

Lời giải: Ta sẽ đi tính giá trị của π∈℘[n] B(π) trước. Quả thực nếu gọi S là tập hợp {1, 2, · · · , n},
P

chúng ta đi đếm số các i với mọi i ∈ S như sau:


n
!
X 1 1
F (x) = Qn j
− Qn j
i=1 j=1 (1 − x ) j=1,j6=i (1 − x )
x
Rút gọn lại ta thu được hệ sinh của π∈℘[n] B(π) là: F (x) = .
P
(1 − x) n j
Q
j=1 (1 − x )
Giờ ta đi tính giá trị của π∈℘[n] A(π), chú ý rằng ta có song ánh giữa các phân hoạch
P

có i số một với i ∈ S và tập hợp n − i phần tử mà không chứa số một nào hết rồi ghép
lại với nhau ta được một phân hoạch của n. Với cách phân hoạch mà không có số một
nào được biểu diễn như sau:
1
f (x) = Qn
j=2 (1 − xj )

Bây giờ với mỗi i, ta có i số một, cho nên hệPsố của x


n−i
trong khai triển f (x) phải
được nhân với i. Từ đó ta có thể tính được π∈℘[n] A(π) như sau:
n
!
X i
ix f (x)
i=1
x
Từ đó dẫn đến hệ sinh của A(π) là: G(x) = f (x) .
P
π∈℘[n]
(1 − x)2
So sánh cách hệ số của F (x) và G(x) ta thu được π∈℘[n] A(π) = π∈℘[n] B(π).
P P


§3 Tính chia hết


Việc phân tích một số thành tích các thừa số nguyên tố thường dễ dàng xử lý những câu hỏi
chia hết có dạng như sau: một số chia hết cho số khác nếu và chỉ nếu lũy thừa mỗi thừa số
nguyên tố trong phân tích thành nhân tử của số thứ nhất lớn hơn hoặc bằng lũy thừa của
số nguyên tố đó trong phân tích thành nhân tử của số thứ hai, hoặc dạng tìm ước chung
lớn nhất và bội chung lớn nhất. Tuy nhiên nội dung về tính chia hết không chỉ đơn giản ở
đó, cần rất nhiều kỹ thuật chứng minh quan trọng và các tính chất chia hết căn bản trong
số học.
Bài toán 1: Tìm số nguyên dương nhỏ nhất K sao cho với mọi tập con K phần tử của {1, 2, ..., 20}
luôn chứa 2 phần tử phân biệt a, b sao cho a + b là số nguyên tố.
Lời giải: Ta có câu trả lời là 11.
Ta có mọi tập con của {2, 4, 6, ..., 20} không chứa hai phần tử phân biệt a, b thỏa mãn
a + b là số nguyên tố. Do |{2, 4, 6, ..., 20}| = 10, nên K không thể nhỏ hơn hoặc bằng
10.(vì nếu K < 10 ta lấy 1 tập con của {2, 4, 6, ..., 20} có điều mâu thuẫn). Suy ra
K > 10.

17
Ta đi chứng minh K ≤ 11 : Ta chia tập {1, 2, ..., 20} thành 10 bộ như sau {1, 2, ..., 20} =
({3, 20} ∪ {4, 19} ∪ · · · ∪ {11, 12}) ∪ {1, 2}. Ta có mọi tập con của {1, 2, ..., 20} có ít
nhất 11 phần tử sẽ có 2 phần tử cùng nằm trong 1 bộ trên (nguyên lý Dirichlet), Suy
ra tổng của 2 số đó là số nguyên tố (3 hoặc 23).
Vậy bài toán có đáp số là K = 11. 
Bài toán 2: Một tập hợp S gồm vài số nguyên dương được gọi là xoáy nếu với mọi ba số phân biệt
a, b, c ∈ S, ta đều có a | bc, b | ca, c | ab.(Iberoamerican Olympiad 2013 )
a) Chứng minh rằng với mọi tập hữu hạn các số nguyên dương {c1 , c2 , . . . , cn } là
tập xoáy thì tồn tại vô hạn các số nguyên dương k sao cho tập {kc1 , kc2 , . . . , kcn }
cũng là tập xoáy.
b) Chứng minh rằng cho mọi số nguyên dương n ≥ 3 thì tồn tại tập xoáy gồm n
phần tử và thỏa mãn không tồn tại số nguyên dương nào lớn hơn 1 là ước của n
phần tử trên.
Lời giải: a) Ta chỉ việc chọn như sau k = m · lcm (c1 , c2 , · · · , cn ) với mọi m ∈ Z+ .
1 Qn
b) Xét n số nguyên tố phân biệt pi , 1 ≤ i ≤ n, và chọn ci = pj . Vậy ta có
pi j=1
tập hợp {c1 , c2 , . . . , cn } thỏa mãn đề bài. 
Bài toán 3: Tìm tất cả các số nguyên k ≥ 2 sao cho tồn tại một tập hợp M gồm k số nguyên
dương sao cho tích của các số trong M đều chia hết cho tổng 2 số phân biệt bất kỳ
trong M . (Czech MO 2014-2015)
Lời giải: Xét k ≥ 2 số nguyên dương phân biệt bất kỳ a1 , a2 , . . . , ak . Xét d = 1≤i<j≤k (ai +aj ),
Q

và tậpQM là tập gồm các số x1 = da1 , x2 = da2 , . . . , xk = dak . Ta có tích của các số này
là dk kℓ=1 aℓ , dễ thấy tích này chia hết cho dai +daj = d(ai +aj ) với mọi 1 ≤ i < j ≤ k.

Bài toán 4: (a) Giả sử S là tập vô hạn các số nguyên dương thỏa mãn : Với mọi tập con hữu hạn
khác rỗng T của S, tồn tại 1 số nguyên dương lớn hơn 1 là ước của các phần tử
của T . Chứng minh rằng tồn tại 1 số nguyên lớn hơn 1 sao cho là ước của các
phần tử của S.
(b) Cho k là số nguyên dương cố định. Chứng minh rằng tồn tại 1 tập hợp vô hạn S
sao cho tất cả các tập con T của S mà có tối đa k phẩn tử sẽ có ước chung lớn
hơn 1, nhưng không có số nguyên dương lớn hơn 1 nào là ước của các phần tử
của S
Lời giải: (a) Xét a > 1 là điểm bất kỳ thuộc S. Khi đó a có hữu hạn ước nguyên tố, giả sử
là p1 , · · · , pn . Giả sử phản chứng không tồn tại 1 số nguyên lớn hơn 1 sao cho là
ước của các phần tử của S.
Giả sử với mỗi số nguyên tố pi bất kỳ, tồn tại ít nhất một phần tử trong S không
chia hết cho pi : Khi đó, với mỗi i, ta chọn xi ∈ S thỏa mãn xi không chia hết
cho pi . Dễ chứng minh được T = {a, x1 , · · · , xn } là tập con hữu hạn của S thỏa
mãn ước chung lớn nhất của tất cả các phần tử trong T bằng 1. Mâu thuẫn. Vậy
ta có điều phải chứng minh.
(b) Xét p1 , · · · , pk+1 đôi một nguyên tố cùng nhau. Xét P = p1 p2 · · · pk+1 và Pi = pPi .
Xét S là tập các số nguyên dương thỏa mãn chia hết cho một trong các số Pi và
không chia hết cho P , do đó mỗi phần tử của S chia hết cho đúng một số Pi .
Ta thấy S chứa tất cả các số Pi , do đó không có ước chung nào lớn hơn 1 của tất
cả các phần tử trong S, thỏa mãn điều kiện thứ 2 của bài toán.
Tiếp theo ta xét T là tập con khác rỗng S có nhiều nhất k phần tử. Do mỗi phần
tử của S chỉ chia hết cho đúng 1 số Pi và có k + 1 số Pi , nên có ít nhất một số
Pi , giả sử là P1 mà không có số nào thuộc T là bội của nó. Suy ra mỗi phần tử
của T chia hết cho p1 . Vậy ta có điều phải chứng minh.

Bài toán 5: Chứng minh rằng với mọi số nguyên n ≥ 2, tồn tại 1 tập hợp S gồm n số nguyên sao
cho ab chia hết cho (a − b)2 với mọi a, b ∈ S phân biệt.(USA, 1998)
Lời giải: Ta thực hiện quy nạp theo n.
Xét trường hợp tầm thường n = 2, ta chọn 2 số liên tiếp nhau là thỏa mãn đề bài.
Gọi a1 , a2 , · · · , aj là j số thỏa mãn (am − an )2 | am an ∀ m 6= n.

18
Y
Xét M = (am − an )2 · a21 a22 · · · a2j . Ta nhận thấy tập {M + a1 , M + a2 , . . . , M + aj }
m6=n
cũng thỏa mãn yêu cầu bài toán do với mọi m 6= n, do
(am − an )2 = ((M + am ) − (M + an ))2 | am an

=⇒ ((M + am ) − (M + an ))2 | (M + am )(M + an )


Ta cần thêm 1 số khác nữa vào tập này để thu được bộ có j + 1 phần tử thỏa mãn đề
bài. Thêm M vào tập trên, do ((M + ai ) − M )2 = a2i | M (M + ai ) ∀ i. Khi đó ta có
tập gồm j + 1 phần tử {M, M + a1 , M + a2 , . . . , M + aj } thỏa mãn yêu cầu bài toán.
Theo nguyên lý quy nạp, ta có điều phải chứng minh. 
Bài toán 6: Xét một tập hợp S = {1, 2, ..., 50}. Tìm số nhỏ nhất k thỏa mãn với mọi tập con k
phần tử của S, tồn tại hai phần tử phân biệt a, b thỏa mãn ab chia hết cho a + b.
Lời giải: Gọi d = gcd(a, b), đặt a = dm, b = dn (m, n là các số nguyên dương và (m, n) = 1).
Điều kiện của a, b có thể viết thành: d(m + n) là ước của d2 mn. ⇒ m + n là ước của
dmn.
Hơn nữa, do (m, n) = 1, suy ra m+n √ nguyên tố cùng nhau với m và n, do đó (m+n)|d.
Do a + b ≤ 99 ⇒ m + n ≤ 99, suy ra m + n ≤ 9. Ta thấy m + n ≥ 1 +
2 = 3 (a, b phân biệt), do đó ta xét từng trường hợp có thể của các bộ (a, b):
(6, 3); (12, 6); (18, 9); (24, 12); (30, 15); (36, 18); (42, 21); (48, 24);
(12, 4); (24, 8); (36, 12); (48, 16)
(20, 5), (40, 10), (15, 10), (30, 20), (45, 30)
(30, 6)
(42, 7), (35, 14), (28, 21)
(40, 24)
(45, 36)
Có 23 bô và 24 số. Gọi G là tập của 26 còn lại. Suy ra, k > 26 (nếu không, chọn ra 1
tập con có các phần tử thuộc G không thỏa mãn đề bài!). Ta cần tìm giá trị nhỏ nhất
của k − 26 thỏa mãn với mọi bộ của k − 26 được chọn từ 24 số ở trên sẽ có ít nhất 1
bộ được liệt kê bên trên. Theo định lý Pigeonhole,k − 26 ≥ [ 24 2
] + 1 = 13 do đó giá trị
nhỏ nhất của k là 39. 
Bài toán 7: Tìm tất cả các tập hợp A = {a1 , a2 , ..., an ; n > 2012} gồm những số nguyên và có tính
chất sau: 2012 ∈ A, đồng thời mỗi tập con tùy ý gồm 2012 số thuộc A đều có thể chia
thành 4 nhóm có số phần tử bằng nhau và tổng các phần tử trong mỗi nhóm bằng nhau.

Lời giải: Điều kiện của bài toán cho thấy rằng tổng của 2012 số tùy ý thuộc A là một số chia
hết cho 4. Thay từ tập hợp 2012 phần tử một phần tử bất kỳ bởi một phần tử khác
tùy ý không thuộc tập đã chọn, tính chất tổng chia hết cho 4 không hề thay đổi. Như
vậy có thể thấy ngay rằng, mọi phần tử thuộc A đều đồng dư nhau modulo 4.
Dĩ nhiên ta nẩy ra ngay dự đoán rằng, các phần tử của A đều bằng nhau. Để có
thể kiểm chứng dự đoán này, ta xét phần tử nhỏ nhất của A là a. Khi đó tập hợp
B = a1 − a, a2 − a, ..., an − a rõ ràng cũng thỏa mãn những tính chất đã nêu trong
bài toán như đối với tập A. Do đó, mọi phần tử của B cũng đồng dư nhau modulo 4,
và vì B chứa phần tử bằng 0 nên suy ra mọi phần tử của B đều chia hết cho 4.
Từ đây, suy luận theo quy nạp lùi quen thuộc cho ta lời giải: tập hợp nhận được từ B
b
bằng cách thay mọi phần tử b ∈ B bởi cũng có tính chất nêu trong bài ra; và do đó
4
các phần tử của tập hợp này cũng chia hết cho 4. Tiếp tục quá trình, dễ suy ra mọi
phần tử của B đều bằng 0. Như vậy mọi số thuộc A đều bằng 2012. 

Bài toán 8: Với mỗi số nguyên dương d, gọi f (d) là số nguyên dương nhỏ nhất có đúng d ước số
dương. Chứng minh rằng với mọi k ≥ 0, f (2k+1 ) chia hết cho f (2k ).

Lời giải: Để giải bài này, rõ ràng ta cần biết những số nào sẽ có 2k ước dương, sau đó mới tìm
số nhỏ nhất trong những số như vậy. Lẽ tự nhiên là ta nghĩ ngay đến việc phân tích
một số nguyên dương n ra thừa số nguyên tố. Ta có phân tích của n:
n = Πp pα (p)
trong đó α(p) là các số nguyên dương.

19
Gọi d(n) là số ước của n thì ta có:
d(n) = Πp (α(p) + 1)
Như vậy, d(n) là một lũy thừa của 2 khi và chỉ khi với mọi p, tồn tại b(p) ≥ 0 sao cho
α(p) = 2b(p) − 1
Ta viết lại cấu trúc của α(p) dưới dạng sau
α(p) = 1 + 21 + ... + 2b(p)−1
b(p)−1 i
n = Πp Πi=1 p2
Khi đó d(n) = 2k , với k = Σp b(p). Như vậy các số n cần tìm là tích của những số nào
r
đó có dạng p2 , với p là số nguyên tố và r nguyên không âm. Từ phân tích trên ta
r m
thấy rằng nếu số p2 có mặt trong biểu diễn n dưới dạng tích, thì những số p2 với
m < r cũng có mặt trong tích đó. Nghĩa là nếu một số nào đó có mặt trong biểu diễn
n thì mọi ước số của nó cũng đều tham gia trong biểu diễn. Do đó ta có kết luận sau
r
đây: nếu gọi S là tập hợp các số dạng p2 với p là số nguyên tố và r nguyên không âm,
thì d(n) là một lũy thừa của 2 khi và chỉ khi n là tích các phần tử thuộc một tập con
hữu hạn T của S có tính chất sau: với mọi t ∈ T, s ∈ S, mà s|t thì s ∈ T . Hơn nữa,
nếu d(n) = 2k thì tập hợp T gồm k phần tử.
Dễ thấy rằng với mọi k nguyên dương, tập hợp Tk gồm k phần tử nhỏ nhất của S
thỏa mãn tính chất trên, suy ra f (2k ) chính là tích các phần tử thuộc Tk . Từ đó suy
ra ngay kết luận của bài toán. 
Chú ý 3.0.2. có thể thấy S = 2, 3, 4, 5, 7, 9, 11, 13, 16, 17, ..., suy ra f (2) = 2, f (4) =
2.3 = 6, f (8) = 2.3.4 = 24, f (16) = 2.3.4.5 = 120, f (32) = 2.3.4.5.7 = 840, ...

Bài toán 9: Cho 81 số nguyên dương phân biệt sao cho các ước nguyên tố của chúng thuộc tập
{2, 3, 5}. Chứng minh rằng tồn tại 4 số trong 81 số trên mà tích của chúng là lũy thừa
bậc 4 của 1 số nguyên nào đó.
Lời giải: Ta có mỗi số nguyên dương của bài có thể biểu diễn dưới dạng 2αi · 3βi · 5γi . Xét đồng
dư αi , βi , γi modulo 2. Ta có mỗi αi , βi , γi có thể có 2 số dư khác nhau modulo 2, do
đó có thể có 2 · 2 · 2 = 8 dạng khác nhau của các lũy thừa này. Theo nguyên lý Dirichle,
có 2 số có cùng dạng số mũ, vì 81 8
> 1. Ta xét tích của 2 số này và đặt tích đó là a1
xóa 2 số trên đi. Ta tiếp tục làm như vậy thu được ai tương tự cho đến khi chỉ còn 8

số còn lại trong dãy . Khi đó ta thu được 81−9 2
= 36 bộ như vậy. Ta thấy các số ai -
là số tự nhiên vì ai là số chính phương. (Và ta lại thấy số mũ của các số có cùng dạng
√ √
số mũ theo modulo 2. Theo nguyên lý Dirichle, ra có 2 số am và an có cùng dạng
số mũ theo modulo 2, vì 36 8
> 1. Xét tích của am và an và ta được lũy thừa bậc 4, vì
chúng cùng là số chính phương và cùng dạng số dư modulo 2 của số mũ. Ta có điều
phải chứng minh. 
Chú ý 3.0.3. Ta có thể giới hạn tốt hơn là số 25 (thay vì 81 trong bài).
Bài toán 10: Cho tập M gồm 1985 số nguyên dương phân biệt, sao cho ước nguyên tố của 1985 số
trên không lớn hơn 23. Chứng minh rằng M chứa 1 tập con gồm có 4 phần tử mà tích
của chúng là lũy thừa bậc 4 của 1 số nguyên nào đó.
Lời giải:
Cách 1: Ban đầu ta chỉ đi tìm 2 số mà sao cho tích của chúng lập thành một số chính
phương. Tổng cộng ta có 9 số nguyên tố, cho nên tối đa có 29 số có các dạng mũ
khác nhau theo modulo 2, nếu ta nhặt ra 29 + 1 số bất kỳ sẽ tồn tại 2 số có cùng
dạng mũ theo modulo 2 và rõ ràng là tích của chúng lập lên số chính phương, bỏ
đi 2 số này trong dãy, ta chỉ quan tâm đến dãy 1983 số còn lại. Tiếp tục nhiều
lần như vậy ta có thể tìm ra (1985 − (29 + 1))/2 ≥ 29 + 1 cặp số mà có tích là số
chính phương. Cho nên ta sẽ lập được 29 + 1 số chính phương, và tương tự như
bài toán trước ta lại tìm ra 2 số mà tích của chúng là lũy thừa bậc 4 của một số
tự nhiện. Ta đã chỉ ra được một tập con gồm 4 phần tử mà tích của chúng là lũy
thừa bậc bốn của một số nguyên nào đó.
Chú ý 3.0.4. Thực tế ta chỉ cần tới 1537 số nguyên dương phân biệt của tập M
là đã đủ giải quyết bài toán. Điều này được chứng minh ở cách làm sau:

20
Cách 2: Các số có ước nguyên tố ≤ 23 có dạng n = 2a1 · 3a2 · . . . · 23a9 , ta có thể đồng
nhất nó với véc tơ v = (a1 (mod 4), . . . , a9 (mod 4)) ⊂ Z94 , với Z4 là nhóm hoán
vị có bậc 4. Ta phải chứng minh với 1985 phần tử đã cho bất kỳ của Z94 , we can
find 4 mà có tổng bằng 0.
Ta ký hiệu mệnh đề dướng đây là Pk (n), với n là một số nguyên dương ≥ 2 và k
cũng là số nguyên dương.
"Cho 2k n − (2k − 1) phần tử của Zkn , ta luôn tìm được n số trong đó có tổng bằng
0."
Ta sẽ chứng minh dạng sau Pk (a), Pk (b) ⇒ Pk (ab). Khi đó, do Pk (2) dễ chứng
minh dựa vào nguyên lý Dirichlet, ta suy ra Pk (4) đúng và điều chúng ta muốn
là P9 (4), chính là điều phải chứng minh.
Giả sử Pk (a), Pk (b) đúng và xét 2k ab − (2k − 1) số nguyên dương bất kỳ. Chọn
ra một nhóm có 2k a − (2k − 1) phần tử và từ đó chọn ra được a phần tử mà
có tổng chia hết cho a. Xóa đi a phần tử này trong dãy ta thu được dãy có
2k ab − (2k − 1) − a phần tử còn lại. Và tiếp tục quá trình này cho đến khi ta có
2k b − (2k − 1) nhóm phân biệt có a phần tử, mà tổng các phần tử mỗi nhóm chia
hết cho a. Tiếp tục chia 2k b − (2k − 1) tổng này cho a, và từ nhóm có 2k b − (2k − 1)
phần tử này ta lấy ra được b phần tử có tổng chia hết cho b. Nhân lại tổng này
với a, và ta thu được 1 nhóm có ab phần tử từ b tổng. Rõ ràng là nhóm có ab
phần tử này có tổng chia hết cho ab. Ta kết thúc quy nạp. 

Bài toán 11: Cho S là tập hữu hạn các số nguyên, mỗi số lớn hơn 1. Giả sử, với mỗi số nguyên n,
tồn tại s ∈ S thỏa mãn ước chung lớn nhất của s và n bằng 1 hoặc s. Chứng minh
rằng tồn tại s, t ∈ S sao cho ước chung lớn nhất của chúng là số nguyên tố. (Putnam,
1999)
Lời giải: Xét số nguyên dương nhỏ nhất k thỏa mãn k không nguyên tố cùng nhau đôi một với
tất cả các phần tử của S (tồn tại k vì lấy bội chung nhỏ nhất của tất cả các phần
tử của S ta có thể tìm được số nhỏ nhất đó). Nếu tồn tại n ∈ S là số nguyên tố ta
có điều phải chứng minh bằng cách xét gcd(n, n). Nếu không tồn tại n ∈ S nào là số
nguyên tố, lấy một ước nguyên tố p|n sao cho kp là số nguyên và dễ thấy là số nhỏ hơn
k. Khi đó gcd(n, k) = n theo giả thiết. Vì do giả sử của k như trên, tồn tại n′ của S
mà kp nguyên tố cùng nhau với n′ . Điều này chỉ xảy ra nếu vp (k) = 1. Dễ thấy p|n′ .
Do gcd(n′ , k) > 1 nên gcd(n′ , k) = p vậy gcd(n′ , n) = p . Ta có điều phải chứng minh.

Bài toán 12: Chứng minh rằng tồn tại tập A gồm những số nguyên dương sao cho cho thỏa mãn
tính chất sau: Với mọi tập vô hạn S gồm những số nguyên tố, tồn tại một số nguyên
dương k ≥ 2 và 2 số nguyên dương m ∈ A và n 6∈ A sao cho m, n là tích của k phần
tử phân biệt của S (IMO, 1994).
Lời giải:
Tập A là tập các số nguyên dương được xác định như sau: một tích n của k số
nguyên tố phân biệt p1 , . . . , pk là thuộc A khi và chỉ khi if n ≡ 1 (mod k + 1). Ví dụ:
p1 · . . . · pk ∈ A ⇔ p1 · . . . · pk ≡ 1 (mod k + 1) với mọi k và mọi số nguyên tố pi .
Gọi S là tập vô hãn các số nguyên tố và gọi p1 là số nguyên tố nhỏ nhất và p2 là số
nguyên tố nhỏ thứ hai thuộc S. Gọi p là số nguyên tố thỏa mãn p > p2 .
Xét S modulo p. Theo nguyên lý Dirichlet, các số dư modulo p có thể xuất hiện vô
hạn lần trong S. Chẳng hạn: tồn tại số nguyên a, sao cho tồn tại vô số q ∈ S mà q ≡ a
(mod p). Rõ ràng, p ∤ a.
Chọn p − 1 số nguyên tố phân biệt q1 , . . . , qp−1 ∈ S và đặt m = q1 · . . . · qp−1 . Khi đó
m ∈ A vì theo định lý Fecma nhỏ, m = q1 · . . . · qp−1 ≡ ap−1 ≡ 1 (mod p).
Mặt khác, ta có p > p2 > p1 , do đó p2 6≡ p1 (mod p). Suy ra, có ít nhất 1 số trong các
số p1 · ap−2 và p2 · ap−2 không đồng dư 1 modulo p. Suy ra, có ít nhất một số trong
các số n1 = p1 · q1 · . . . · qp−2 và n2 = p2 · q1 · . . . · qp−2 không đồng dư 1 modulo p và
suy ra nó không thuộc A. 
Bài toán 13: Cho S = {1, 2, 3, . . . , 280}. Tìm số nguyên dương n bé nhất sao cho với mọi tập con n
phần tử của S chứa 5 số mà đôi một nguyên tố cùng nhau. (IMO, 1991)

21
Lời giải: Gọi A2 - tất cả các số thuộc S,- và chia hết cho 2; A3 - tất cả các số thuộc S,- và chia
hết cho 3 và không thuộc A2 ; A5 - tất cả các số thuộc S, và chia hết cho 5 và không
thuộc A2 , A3 ; A7 - tất cả các số thuộc S, chia hết cho 7 and không thuộc A2 , A3 , A5 .
|A2 | = 140, |A3 | = 47, |A5 | = 19, |A7 | = 10
Gọi Ap - tất cả các số nguyên tố thuộc (7, 280), và An - tất cả các số, không thuộc
A2 , A3 , A5 , A7 , Ap |Ap | = 56. Rõ ràng An = {11.11, 11.13, 11.17, 11.19, 11.23, 13.13, 13.17, 13.19}, |An | =
8 và S = A2 ∪ A3 ∪ A5 ∪ A7 ∪ Ap ∪ An .
Ta có 2 tập bất kỳ Ax (x = 2, 3, 5, 7, p, n) không có phần tử chung. Ta xây dựng tập
A = A2 ∪ A3 ∪ A5 ∪ A7 ; |A| = 140 + 47 + 19 + 10 = 216. Lấy ngẫu nhiên 5 số từ A.
Theo nguyên lý Dirichle, có hai số ta lấy được từ một tập Ax (x = 2, 3, 5, 7). Rõ ràng
2 số này không nguyên tố cùng nhau. Do đó n ≥ |A| + 1 = 217
Ta đi chứng minh mọi tập B - tập con của S và |B| = 217 thì B chứa 5 số nguyên tố
cùng nhau đôi một.
Gọi P là tập các số chứa 1 và các số nguyên tố nhỏ hơn 280. Suy ra |P | = 60. Xét các
tập sau:
A1 = {2.41, 3.37, 5.31, 7.29, 11.23, 13.19};
A2 = {2.37, 3.31, 5.29, 7.23, 11.19, 13.17};
A3 = {2.31, 3.29, 5.23, 7.19, 11.17, 13.13};
A4 = {2.29, 2.23, 5.19, 7.17, 11.13};
A5 = {2.23, 3.19, 5.17, 7.13, 11.11};
⇒ |P ∪ A1 ∪ · · · ∪ A5 | = 88
B chứa ≥ 280 − 217 + 88 = 25 phần tử thuộc P ∪ A1 ∪ · · · ∪ A5 . Nếu tồn tại 5 phần
tử thuộc cùng một tập P hoặc Ai với i = 1, 2, . . . , 5 thì ta thu được 5 số đôi một
nguyên tố cùng nhau. Nếu không, ta chỉ chọn tối đa được 4.6 = 24 phần tử. Vô lý .
Vậy n = 217. 
Bài toán 14: Xét tập hợp S = {1, 2, 3, · · · , 6n}, n > 1. Tìm giá trị lớn nhất của k sao cho khẳng
định sau là đúng: với mọi tập con A của S chứa 4n phần tử có ít nhất k cặp (a, b),
a < b và b chia hết cho a.(Brazilian 2015)
Lời giải: Lemma: Cho n ∈ N và gọi S là tập con có n + 1 phần tử của {1, 2, · · · 2n}. Khi đó S
có 2 phần tử thỏa mãn một số chia hết cho số còn lại.
Chứng minh: Phân tích {1, 2, · · · , 2n} tạo thành các tập con có dạng {p · 2k | k ∈ N0 }
với số nguyên tố p ≤ 2n. Do ta đã phân tích tập ban đầu thành π(2n) ≤ n tập con,
theo nguyên lý Dirichle, tồn tại 2 phần tử của S phải thuộc một trong những tập con
này. Khi đó một trong những phần tử này chia hết cho số còn lại, điều phải chứng
minh. 
Quay trở lại bài toán, xét A = {2n + 1, 2n + 2, · · · , 6n}. Nếu x, y ∈ A là các số phân
biệt và thỏa mãn x | y, và do xy < 3, suy ra y/x = 2. Khi đó, ta dễ chia tất cả các bộ
(x, y) ∈ A2 thỏa mãn x | y và x 6= y như sau: mỗi bộ đều có dạng (2n + t, 4n + 2t) với
t = 1, 2, · · · , n. Nó kéo theo k ≤ n.
Bây giờ ta sẽ chứng minh k ≥ n. Giả sử ngược lại, và gọi (x1 , y1 ) , (x2 , y2 ) , · · · , (xm , ym ),
với m < n, là tất cả các bộ (x, y) ∈ A2 thỏa mãn x | y and x 6= y. Ta xóa x1 , x2 , · · · , xm
khỏi A và tạo một tập mới B. Rõ ràng là ở tập B không có bộ (x, y) ∈ B 2 nào thỏa
mãn x | y and x 6= y. Tuy nhiên, do |B| ≥ |A| − m ≥ 3n + 1, mâu thuẫn với bổ đề.
Điều này kéo theo k ≥ n , suy ra k = n. . 

§4 Các bài toán về tổng


"Điều gì xảy ra khi các tập các sô nguyên được cộng vào nhau?" là một trong những câu
hỏi nghiên cứu lớn nhất trong số học-tổ hợp, kết nối tổ hợp với không chỉ mỗi số học mà
còn với cả đại số và giải tích. Một trong những bài toán rất nổi tiếng mà đến nay vẫn chưa
giải được là giả thiết Goldbach mạnh "Tất cả các số chẵn lớn hơn 2 có thể biểu diễn thành
tổng của hai số nguyên tố". Trong mục này, những bài tập dưới đây cung cấp nhưng kỹ
thuật tổ hợp và đại số rất giá trị, điển hình là nguyên lý Dirichlet, nguyên lý cực hạn, đếm
bằng nhiều cách...

22
Bài toán 1: Chứng minh rằng bất cứ tập hợp n số nguyên chứa tập con khác rỗng mà tổng các
phần tử của chúng chia hết cho n.
Lời giải: Xét các số nguyên x1 , . . . , xn , và xét các tổng thành phầnx1 , x1 + x2 , x1 + x2 +
x3 , . . . , x1 + x2 + · · · + xn lập được từ những số đó. Giả sử không có tổng nào chia hết
cho n, thì phải tồn tại 2 tổng có cùng số dư. Vậy hiệu của chúng chia hết cho n. 
Bài toán 2: Cho A là tập con của {0, 1, 2, . . . , 1997} chứa nhiều hơn 1000 phần tử. Chứng minh
rằng A chứa hoặc lũy thừa của 2, hoặc chứa 2 phần tử phân biệt sao cho tổng của
chúng là lũy thừa của 2. (Ireland, 1997)
Lời giải: Giả sử A không thỏa mãn yêu cầu nào của bài toán. Xét 996 bộ dời nhau:
1. tổng là 2048: (1997, 51), (1996, 52), . . . , (1025, 1023)
2. tổng là 64: (50, 14), (49, 15), (48, 16), . . . , (33, 31)
3. tổng là to 16: (13, 3), (12, 4), (11, 5), . . . , (9, 7)
Ta chỉ chọn được nhiều nhất một số từ mỗi cặp. Và ta phải tránh các số là lũy thừa
của 2 ,vì vậy ta phải chọn ít nhất 4 số không thuộc 996 bộ trên. Khi đó ta chọn 0. Tất
cả còn lại chưa được đưa vào bộ là 1, 2, 8, 32, 1024, và nếu một trong các số này được
chọn thì ta có mâu thuẫn với giả sử. Do đó tập A có nhiều nhất 997 phần tử. Ví dụ
nếu ta chọn A = {1997, 1996, . . . , 1025, 50, 49, 48, . . . , 33, 13, 12, 11, . . . , 9, 0} thì ta có
tập 997 phần tử thỏa mãn yêu cầu bài toán. Vậy giả sử là sai, ta có điều phải chứng
minh. 
Sau đây là bài thi tuyển chọn vòng 2 TST của Poland với dạng bài trên nhưng câu
hỏi mở hơn như sau:
Bài toán 3: Cho X là tập hợp {1, 2, 3, ..., 2001}. Tìm số n nhỏ nhất sao cho với mọi tập con n phần
tử của X ta luôn tìm được phần tử là lũy thừa của 2 hoặc ta tìm được 2 phần tử mà
tổng của chúng là lũy thừa của 2. (Poland TST 2001)
Lời giải: Xét 998 cặp sau:
(2, 14), . . . , (7, 9)
(15, 17), (18, 46), (19, 45), . . . , (31, 33)
(47, 2001), (48, 2000), . . . , (1023, 1025)
và bộ các số còn lại là 1, 8, 16, 32, 1024.
Với tập con S có 999 phần tử của X, nếu có ít nhất một số trong 1, 8, 16, 32, 1024 ∈ S
thì ta có điều phải chứng minh. Ngược lại, theo nguyên lý Dirichle, tồn tại một cặp
trong 999 tập liệt kê ở trên thuộc S. Với n = 998 ta chọn tập con {9, 10, . . . , 14} ∪
{17} ∪ {33, 34, . . . , 46} ∪ {1025, 1026, . . . , 2001} đây là ví dụ không thỏa mãn đề bài.
Vậy n = 999 là giá trị nhỏ nhất cần tìm. 
Bài toán 4: 50 số nguyên được chọn từ tập hợp {1, . . . , 99}, sao cho không có hai số nào có tổng
bằng 99 hoặc 100. Chứng minh rằng các số được chọn phải là 50, 51, 52, . . . , 99. (St.
Petersburg, 1997)
Lời giải: Nhóm các số nguyên 1, . . . , 100 thành 49 bộ {x, 100 − x} with x ∈ {1, . . . , 49}, và còn
lại một số 50
Nếu ta chọn đồng thời hai số nguyên từ một trong 49 bộ đầu tiên, thì tổng của chúng
là 100. Cho nên mỗi cặp chỉ được lấy chính xác 1 số. Theo đầu bài ta cần lấy 50 số từ
tập {1, . . . , 99} nên ta buộc phải chọn số 50. Do không có 2 số nào có tổng là 99 nên từ
cặp {49, 51} ta lấy ra số 51. Tương tự ta chỉ có thể chọn được các số sau 52, 53, . . . , 99.
Ta có điều phải chứng minh.
Chú ý 4.0.5. Với bài toán trên thì điều kiện không có tổng hai số nào bằng 99 lỏng
hơn. Ta có thể bỏ điều kiện này thì sẽ cho ta bài toán sau :
"50 số nguyên chọn ra từ 100 số tự nhiên đầu tiên sao cho không có 2 số nguyên nào
có tổng là 100. Chứng mỉnh rằng tồn tại ít nhất 1 số chính phương trong các số được
chọn trên."
Cũng với cách chia trên thì ta buộc chọn 1 số trong cặp {36, 64} vậy ta có điều phải
chứng minh.
Bài toán 5: Cho một dãy n số nguyên dương mà tổng của chúng nhỏ hơn 2n. Chứng minh rằng

23
với mọi số nguyên dương m không vượt quá tổng của n số nguyên dương trên, chúng
ta có thể chọn ra vài số nguyên sao cho tổng của chúng bằng m.
Lời giải: Ta quy ước tập rỗng có tổng bằng 0.
Ta sẽ chứng minh bằng quy nạp. Dễ thấy trường hợp n = 1 đúng.
Giả sử bài toán đúng với mọi số nhỏ hơn n. Ta xét 2 trường hợp sau:
Trường hợp 1: mọi số nguyên trong dãy đều là 1 hoặc 2.
Khi đó ta tiếp tục chọn 2 cho đến khi ta đạt được m, nếu không thì sẽ đạt được m − 1
công việc còn lại là ta chọn thêm số 1 (trong dãy phải có ít nhất một số bằng 1 vì nếu
ngược lại tổng của dãy bằng 2n - mâu thuẫn với giả thiết) hoặc nếu ta chọn hết các
số 2 trong dãy mà tổng nhỏ hơn m − 1 thì ta chọn tiếp đến các số 1’ cho đến khi có
được tổng bằng m.
Trường hợp 2: tồn tại số nguyên k > 2.
Khi đó ta xóa số nguyên này khỏi dãy đã cho và khi đó ta có được dãy n − 1 số nguyên
với tổng bằng S − k là một số nhỏ hơn 2n − k < 2(n − 1). Nếu m ≤ S − k, áp dụng giả
thiết quy nạp ta có điều phải chứng minh. Xét trường hợp S − k < m ≤ S, khi đó ta
có một dãy con có tổng bằng m − k (giả thiết quy nạp) và ta sẽ cộng thêm số bị xóa
k. (Ta cũng phải chứng minh m − k ≥ 0 trong trường hợp này, tuy nhiên điều này dễ
thấy từ k ≤ n và S − k ≥ n − 1). 
Bài toán 6: Chứng minh rằng từ 1 tập 10 phần tử gồm các số có hai chữ số, có thể chọn được
hay không 2 tập con khác rỗng rời nhau mà tổng các phần tử của chúng bằng nhau.
(IMO, 1972)
Lời giải: Ta thấy tổng lớn nhất có thể của các phần tử trong 1 tập con có 10 phần tử là
91 + 92 + ... + 100 = 955. Ta có 210 − 1 = 1023 tập con khác rỗng của mọi tập có 10
phần tử, suy ra theo nguyên lý Dirichle, ta có 2 tập con có cùng giá trị tổng các phần
tử. Gọi các tập con A và B, và xét giao của 2 tập này là A ∩ B. Nếu A ∩ B = ∅, ta
được 2 tập rời nhau A và B, ta có điều phải chứng minh. Nếu A ∩ B 6= ∅, xét các tập
A − (A ∩ B) và B − (A ∩ B). Hai tập này là rời nhau, và chúng có cùng tổng, nên ta
có điều phải chứng minh. 
Bài toán 7: Tìm số nguyên dương lớn nhất n sao cho tồn tại n số nguyên không âm x1 , x2 , . . . , xn ,
thỏa mãn với mọi dãy các phần tử ǫ1 , ǫ2 , . . . , ǫn thuộc {−1, 0, 1}, không cùng bằng 0,
n3 không là ước của ǫ1 x1 + ǫ2 x2 + · · · + ǫn xn . (Romania, 1996)
Lời giải: Ta chỉ lấy các ǫi thuộc tập {0, 1} , tất cả các số được lập từ dãy này ta lấy phần dư
của chúng khi chia cho n3 , trừ chúng với nhau ta được một số có thể biểu diễn bởi
ǫi ∈ {−1, 0, 1}và sẽ chia hết cho n3 nếu tồn tại 2 số dư giống nhau.
Suy ra 2n ≤ n3 , suy ra 2 ≤ n ≤ 9. Ta cần chứng minh n = 9 thỏa mãn yêu cầu bài
toán. Với x1 = 1, x2 = 2, x3 = 4, ..., x9 = 28 = 256 dễ thấy là dãy thỏa mãn yêu cầu
bài toán, do đó ta có n = 9 là giá trị cần tìm. 
Bài toán 8: Cho số nguyên n > 2. Tìm số nguyên lớn nhất d, sao cho từ mọi tập S gồm n số
nguyên, ta có thể tìm được 3 tập con khác nhau (khác rỗng và không nhất thiết dời
nhau), thỏa mãn tổng các phần tử của mỗi tập con đó chia hết cho d. (Czech and
Slovak, 2015)
Lời giải: Xét S = {n + 1, 2n + 1, . . . , n2 + 1} thì để thỏa mãn yêu cầu bài toán ta phải có d < n,
do chỉ có tập con khác rỗng có tổng các phần tử của nó chia hết cho n là chính tập S.
Ta khẳng định d = n − 1, bởi sử dụng 2 bổ đề quen thuộc sau.
Bổ đề 1. Với một tập đã cho có (ít nhất) N (phân biệt) số nguyên, tồn tại một tập
con khác rỗng có tổng các phần tử chia hết cho N . Đây là kết quả Erdös đã biết.
Bổ đề 2. Chỉ có duy nhât tập gồm N − 1 (phân biệt) số nguyên, mà không có tập con
khác rỗng nào của nó chia hết cho N , là những số nguyên đồng dư theo modulo N , và
nguyên tố cùng nhau với N . Bổ đề này khá quen thuộc, với lời giải khá sơ cấp.
Theo bổ đề 1, tồn tại tập con ∅ = 6 A ⊆ S có tổng các phần tử chia hết cho n − 1. Lấy
tùy ý a ∈ A. Lại theo bổ đề 1, tồn tại tập con ∅ 6= B ⊆ S \ {a} có tổng các phần tử
chia hết cho n − 1, và rõ ràng B 6= A. Lấy tùy ý b ∈ B, suy ra b 6= a. Nếu tồn tại một
tập con ∅ 6= C ⊆ S \ {a, b} có tổng các phần tử chia hết cho n − 1, ta có điều phải
chứng minh. Ngược lại, theo bổ đề 2, tồn tại r nguyên tố cùng nhau với n − 1 thỏa

24
mãn x ≡ r (mod n − 1) với mọi x ∈ S \ {a, b}.
Nếu a ≡ 0 (mod n − 1) hoặc b ≡ 0 (mod n − 1) ta dễ có điều phải chứng minh, do đó
ta chỉ cần xét các trường hợp còn lại. Nhận xét này cũng giải quyết trường hợp n = 3,
do đó ta có thể giả sử n > 3.
Có B là tập được tạo bởi b và một số phần tử trong S \ {a, b}. Nếu không phải tất cả
các phần tử trong S \ {a, b} đều được sử dụng, thì ta có thể dễ có tập B ′ bằng cách
đổi một số phần tử (sao cho B ′ khác B và A) có tổng các phần tử chia hết cho n − 1.
Chúng ta chỉ còn trường hợp B = S \ {a}, với b ≡ r (mod n − 1). Do A là tập được
tạo bởi a và các số khác trong n − 1 phần tử còn lại, mà các phần tử này đều đồng
dư với r modulo n − 1. Do không phải tất cả các phần tử đều được sử dụng nên ta có
thể đổi phần tử đê thu được các tập A′ và A′′ (khác với A) có tổng các phần tử chia
hết cho n − 1. 
Bài toán 9: Cho x1 , x2 , . . . , x19 là các số nguyên dương nhỏ hơn hoặc bằng 93. Cho y1 , y2 , . . . , y93
là các số nguyên dương nhỏ hơn hoặc bằng 19. Chứng minh rằng tồn tại tổng của một
vài (khác rỗng) xi bằng tổng của một vài yj . (Putnam, 1993)
Lời giải: Giả sử mâu thuẫn. Không mất tính tổng quát, giả sử i xi . Với mỗi tổng
P P
i yi >
thành phần của các số x, xét các tổng thành phần của các số y có tổng lớn hơn mà
có số phần tử ít nhất. Sự khác nhau giữa các y-tổng vàx-tổng thuộc {1, . . . , 18}. (18
vì ta giả sử không thể có các tổng bằng nhau.) Nhưng có 19 tổng thành phần của
x, do đó được lặp lại. Giả sử ji=1 x = ji=1 y và ji=1
Pj4
i=1 yi và ( giả sử
P1 P2 P3
xi =
Pj3 i Pj4 i
j1 < j3 , j2 < j4 ). Từ đó suy ra j1 +1 xi = j2 +1 yi . Điều phải chứng minh. 
Bài toán 10: Hỏi có bao nhiêu tập con của tập hợp {1, 2, 3, ...., 2000} mà tổng các phần tử của nó
chia hết cho 5?
Lời giải: Xét f (x) = 2000
P2001000
k
aj xj = 1 + x + x2 + 2x3 + · · · + x2001000 . Khi đó
Q
k=1 (1 + x ) = j=0
tổng cần tính là a0 + a5 + · · · + a2001000 (bao gồm cả tập ∅). Ta cần tính f (ω m ) với ω
là nghiệm của phương trình x5 = 1 và m ∈ {0, 1, 2, 3, 4}.
Đặt A0 = a0 +a5 +· · ·+a2001000 , A1 = a1 +a6 +· · ·+a2000996 , A2 = a2 +a7 +· · ·+a2000997 ,
A3 = a3 + a8 + · · · + a2000998 , A4 = a4 + a9 + · · · + a2000999 .
Khi đó 2400 = f (ω) = A0 + A1 ω + A2 ω 2 + A3 ω 3 + A4 ω 4 , 2400 = f (ω 2 ) = A0 +
A1 ω 2 + A2 ω 4 + A3 ω + A4 ω 3 , 2400 = f (ω 3 ) = A0 + A1 ω 3 + A2 ω + A3 ω 4 + A4 ω 2 ,
 = A0 +A1 +A2 +A3 +A4 .
2400 = f (ω 4 ) = A0 +A1 ω 4 +A2 ω 3 +A3 ω 2 +A4 ω, 22000 = f (1)
2402 4799 + 1
Cộng tất cả 5 biểu thức trên ta được A0 = . 
5
Bài toán 11: Cho p là số nguyên tố lẻ. Xác định số tập con có p phần tử của tập hợp {1, 2, . . . , 2p}
sao cho tổng các phần tử chia hết cho p. (IMO, 1995)
Lời giải: Đặt X := {1, 2, ..., p}, Y := {p + 1, p + 2, ..., 2p}. Ta thấy X và Y là hệ đại diện trong
Z/pZ.
Với tập con A ⊂ X và z ∈ Z/pZ đặt A + z := {a + z ∈ X|a ∈ A} và tương tự với tập
con B ⊂ Y .
Gọi A ⊂ X là nghiệm tầm thường khi và chỉ khi A = ∅ or A = X, tương ứng với
B ⊂Y.
Ta chỉ có 2 tập con P ⊂ (X ∪ Y ) thỏa mãn đồng thời P ∩ X và P ∩ Y là tầm thường,
do đó gọi nó là các tập con tầm thường.
Khi đó với mọi P ⊂ (X ∪ Y ) ta có phép tịnh tiến như sau: nếu P ∩ X không phải là tập
con tầm thường thì P + z := ((P ∩ X) + z) ∪ (P ∩ Y ) và P + z := (P ∩ X) ∪ ((P ∩ Y ) + z)
trong trường hợp ngược lại.
Dễ thấy khi z chạy khắp hệ thặng dự modulo p, thì sẽ tồn tại chính xác một tổng
trong số các P + z chia hết cho p. Do các phép tình tiến này chia các tập con không
tầm thường ra thành các lớp tương đương, mỗi lớp có p tập, tất các các tập con ở
cùng một lớp  có các phần tử giống nhau.
Vì ta có 2pp
tập con có sô phần tử là p và 2 trong số chúng là nghiệm tầm thường
(2p ) −2
nên ta có p p + 2 tập con thỏa mãn. 
Bài toán 12: Cho 2p − 1 số nguyên với p là số nguyên tố , chứng minh rằng ta có thể chọn được
chinh xác p số trong các số trên sao cho tổng của chúng chia hết cho p.

25
Lời giải: Trước hết ta cần đề cập đến định lý Chevalley-Warning được phát biểu như sau:
Chevalley - Warning theorem. Xét các đa thức fP 1 , ..., fm với các biến x1 , ..., xn
m
hữu hạn trong trường Fq , với q = pk . Giả sử n > i=1 deg fi . Khi đó số nghiệm
(x1 , ..., xn ) của các hệ phương trình f1 (x1 , ..., xn ) = 0,...,fm (x1 , ..., xn ) = 0 chia hết
cho p.
Giải bổ đề. Đầu tiên, Q ta tìm cách đẹp đểq−1 đếm số nghiệm ( mod p):
Gọi g(x1 , ..., xn ) = m i=1 (1 − fi (x1 , ..., xn ) ).
Ta có nhận xét:
* Nếu (x1 , ..., xn ) là nghiệm, khi đó g(x1 , ..., xn ) = 1.
* Nếu (x1 , ..., xn ) không là nghiệm, khi đó g(x1 , ..., xn ) = 0.
Nhận xét đầu tiên là dễ thấy. Bây giờ ta xét nhận xét thứ hai. Do (x1 , ..., xn ) không
là nghiệm, ta có fi (x1 , ..., xn ) 6= 0 với i. Suy ra fi (x1 , ..., xn )q−1 = 1 và ta thấy một
trong tích của g(x1 , ..., xn ) là 0, suy ra ta chứng minh được nhận xét 2.
Suy
P ra ta đếm số nghiệm mod p có cùng đánh giá.
(x1 ,...,xn )∈Fn g(x1 , ..., xn ).
q
Bây
P giờ ta xét mở rộng g của tổngP các đơn thức: P
(x1 ,...,xn )∈Fn g(x1 , ..., xn ) = (x ,...,xn )∈Fn (d1 ,...,dn )∈Nn cd1 ,...,dn xd11 · · · xdnn
q
P 1 q
P 0 .
= (d1 ,...,dn )∈Nn cd1 ,...,dn (x1 ,...,xn )∈Fn xd11 · · · xdnn
q
P 0
Nhận xét: Nếu tồn tại một di < q − 1, thì (x1 ,...,xn )∈Fn xd11 · · · xdnn = 0.
q
d1 Qn P di
Thật vậy, dn
xi ∈Fq xi . Suy ra
d
= 0 for
P P
(x1 ,...,xn )∈Fn x1 · · · xn
q
= i=1 x∈Fq x
d < q − 1. Nó khá là quen thuộc, được suy ra từ đồng nhất thức xq − x = 0 cho x ∈ Fq
từ đó kéo theo tất các đa thức hoán vị trong các phần tử của Fq của bậc lớn nhất q − 2
là 0 và do đó tất P
cả các đa thức hoán vịP của bậc lớn nhất là q − 2.
Suy ra tổng của (d1 ,...,dn )∈Nn cd1 ,...,dn (x1 ,...,xn )∈Fn xd11 · · · xdnn là tổng các hạng tử
0 q
mà d1 , ..., dn ≥Pq − 1. Cho nên bậc của mỗi hạng tử ít nhất là n(q − 1). Mà bậc của g
nhiều nhất là m i=1 (q − 1) deg fi < n(q − 1) theo giả sử. Cho nên cd1 ,...,dn = 0 với tất
cả các hạng tử và chúng ta đã chứng minh xong bổ đề. Xét hệ sau:
xp−1
1 + xp−1
2 + ... + xp−1
2p−1 ≡ 0 mod pa1 xp−1
1 + a2 xp−1
2
p−1
+ ... + a2p−1 x2p−1 ≡0 mod p

gồm các phương trình theo mod p có 2p − 1 ẩn. Do tổng bậc của cả hai phương trình
là (p − 1) + (p − 1) = 2p − 2, ít hơn số biến nên ta có thể áp dụng Chevalley-Warning.
Dễ thấy ta có một nghiệm x1 = x2 = ... = x2p−1 = 0, và số nghiệm của chúng là chia
hết cho p > 1, nên chắc chắn còn nghiệm khác.
Với nghiệm khác bất kỳ khác nghiệm x1 = x2 = ... = x2p−1 = 0, ta đặt yi = xip−1 .
Theo định lý Fermat, yi ≡ 0 mod p ⇐⇒ xi ≡ 0 mod p và yi ≡ 1 mod p trong các
trường hợp còn lại.
Cho nên chúng ta có thể thấy rằng yi ≡ 0, 1 mod p. Theo giả sử các yi không cùng
đồng thời bằng 0, cho nên dựa vào phương trình thứ nhất của hệ là
y1 + y2 + .. + y2p−1 ≡ 0 mod p
cho chúng ta chính xác p các yi ≡ 1 mod p (còn lại thì bằng 0).
Thay chúng vào phương trình thứ hai của hệ
a1 y1 + a2 y2 + .. + a2p−1 y2p−1 ≡ 0 mod p
cho ta thấy tổng của chính xác p số ai sẽ ≡ 0 mod p, hay là chia hết cho p. Ta có
điều phải chứng minh. 
Bài toán 13: Cho 2n − 1 số nguyên , chứng minh rằng ta có thể chọn được chinh xác n số trong các
số trên sao cho tổng của chúng chia hết cho n. (định lý Erdős-Ginzburg-Ziv )
Lời giải: Gọi EGZn nghĩa là mệnh đề của đề bài đúng với n, nghĩa là ta có thể chọn từ tập
2n − 1 (hoặc hơn) số nguyên chính xác n số thỏa mãn tổng của chúng chia hết cho n
. Khi đó nếu có EGZa và EGZb , ta cũng có thể chứng minh EGZab là đúng.
Chứng minh:
Lấy 2ab − 1 số đã cho. Do ta giả sử EGZa là đúng, ta có thể chọn a số của chúng thỏa

26
mãn tổng của chúng s1 chia hết cho a. Khi đó 2a(b − 1) − 1 số còn lại, ta có thể chọn
lại a số trong đó thỏa mãn tổng của chúng s2 chia hết cho a. Tiếp tục cho đến khi còn
lại a − 1 số. Ta tính được ta đã làm 2b − 1 lần a số và cho ta s1 , s2 , ..., s2b−1 tổng. Bây
giờ ta có đúng 2b − 1 tổng, do đó ta có thể áp dụng EGZb và tìm b trong các số si
thỏa mãn tổng của chúng S chia hết cho b. Quay lại thay các tổng si bởi các số ban
đầu, ta có S là tổng của đúng ab số ban đầu. Suy ra tổng này chia hết cho ab. Điều
phải chứng minh. 

27
Tài liệu
[1] Hà Huy Khoái. Một số bài toán Số học - Tổ hợp. Addison-Wesley, Reading, Mas-
sachusetts, 1993.
[2] Herbert S. Wilf. Lectures on Integer Partitions. University of Pennsylvania(2000).
[3] G. H. Hardy and S. Ramanujan. Asymptotic formula in combinatory analysis. London
Math. Soc. 17 (1918), 175–115.
[4] Hans Rademacher. On the partition function p(n). Proc. London Math. Soc. 43 (1937),
241–254.
[5] Art of problem solving Forum,
http://www.artofproblemsolving.com/
[6] Diễn đàn toán học,
http://diendantoanhoc.net/

28

You might also like